pubcorp midterm digests.docx

44
G.R. No. 180206 February 4, 2009 THE CITY GOVERNMENT OF BAGUIO CITY, represented by REINALDO BAUTISTA, JR., City Mayor; THE ANTI-SQUATTING COMMITTEE, represented by ATTY. MELCHOR CARLOS R. RAGANES, CITY BUILDINGS and ARCHITECTURE office, represented by OSCAR FLORES; and PUBLIC ORDER and SAFETY OFFICE, Represented by EMMANUEL REYES, Petitioners. vs. ATTY. BRAIN MASWENG, Regional Officer-National Commission on Indigenous People-CAR, ELVIN GUMANGAN, NARCISO BASATAN and LAZARO BAWAS, Respondents. Petitioners, the City Government of Baguio City, represented by its Mayor, Reinaldo Bautista, Jr., the Anti-Squatting Committee, represented by Atty. Melchor Carlos R. Rabanes; the City Buildings and Architecture Office, represented by Oscar Flores; and the Public Order and Safety Office, represented by Emmanuel Reyes and later substituted by Gregorio Deligero, assail the Decision1 of the Court of Appeals in CA G.R. SP No. 96895, dated April 16, 2007, and its Resolution2 dated September 11, 2007, which affirmed the injunctive writ issued by the National Commission on Indigenous Peoples (NCIP) against the demolition orders of petitioners. The following undisputed facts are culled from the assailed Decision: The case stemmed from the three (3) Demolition Orders issued by the City Mayor of Baguio City, Braulio D. Yaranon, ordering the demolition of the illegal structures constructed by Lazaro Bawas, Alexander Ampaguey, Sr. and a certain Mr. Basatan on a portion of the Busol Watershed Reservation located at Aurora Hill, Baguio City, without the required building permits and in violation of Section 69 of Presidential Decree No. 705, as amended, Presidential Decree No. 1096 and Republic Act No. 7279. Pursuant thereto, the corresponding demolition advices dated September 19, 2006 were issued informing the occupants thereon of the intended demolition of the erected structures on October 17 to 20, 2006. Consequently, Elvin Gumangan, Narciso Basatan and Lazaro Bawas (hereinafter private respondents) filed a petition for injunction with prayer for the issuance of a temporary restraining order and/or writ of preliminary injunction against the Office of the City Mayor of Baguio City through its Acting City Mayor, Reynaldo Bautista, the City Building and Architecture Office, the Anti-Squatting Task Force, and the Public Order and Safety Division, among others, (collectively called petitioners) before the National Commission on Indigenous Peoples, Cordillera Administrative Region (NCIP-CAR), Regional Hearing Office, La Trinidad, Benguet, docketed as Case No. 31-CAR-06. In their petition, private respondents basically claimed that the lands where their residential houses stand are their ancestral lands which they have been occupying and possessing openly and continuously since time immemorial; that their ownership thereof have been expressly recognized in Proclamation No. 15 dated April 27, 1922 and recommended by the Department of Environment and Natural Resources (DENR) for exclusion from the coverage of the Busol Forest Reserve. They, thus, contended that the demolition of their residential houses is a violation of their right of possession and ownership of ancestral lands accorded by the Constitution and the law, perforce, must be restrained. On October 16 and 19, 2006, Regional Hearing Officer Atty. Brain S. Masweng of the NCIP issued the two (2) assailed temporary restraining orders (TRO) directing the petitioners and all persons acting for and in their behalf to refrain from enforcing Demolition Advice dated September 18, 2006; Demolition Order dated September 19, 2006; Demolition Order No. 25, Series of 2004; Demolition Order No. 33, Series of

Upload: connie-beb

Post on 03-Jan-2016

78 views

Category:

Documents


1 download

TRANSCRIPT

Page 1: pubcorp midterm digests.docx

G.R. No. 180206 February 4, 2009THE CITY GOVERNMENT OF BAGUIO CITY, represented by REINALDO BAUTISTA, JR., City Mayor; THE ANTI-SQUATTING COMMITTEE, represented by ATTY. MELCHOR CARLOS R. RAGANES, CITY BUILDINGS and ARCHITECTURE office, represented by OSCAR FLORES; and PUBLIC ORDER and SAFETY OFFICE, Represented by EMMANUEL REYES, Petitioners. vs.ATTY. BRAIN MASWENG, Regional Officer-National Commission on Indigenous People-CAR, ELVIN GUMANGAN, NARCISO BASATAN and LAZARO BAWAS, Respondents.

Petitioners, the City Government of Baguio City, represented by its Mayor, Reinaldo Bautista, Jr., the Anti-Squatting Committee, represented by Atty. Melchor Carlos R. Rabanes; the City Buildings and Architecture Office, represented by Oscar Flores; and the Public Order and Safety Office, represented by Emmanuel Reyes and later substituted by Gregorio Deligero, assail the Decision1 of the Court of Appeals in CA G.R. SP No. 96895, dated April 16, 2007, and its Resolution2 dated September 11, 2007, which affirmed the injunctive writ issued by the National Commission on Indigenous Peoples (NCIP) against the demolition orders of petitioners. The following undisputed facts are culled from the assailed Decision:The case stemmed from the three (3) Demolition Orders issued by the City Mayor of Baguio City, Braulio D. Yaranon, ordering the demolition of the illegal structures constructed by Lazaro Bawas, Alexander Ampaguey, Sr. and a certain Mr. Basatan on a portion of the Busol Watershed Reservation located at Aurora Hill, Baguio City, without the required building permits and in violation of Section 69 of Presidential Decree No. 705, as amended, Presidential Decree No. 1096 and Republic Act No. 7279.Pursuant thereto, the corresponding demolition advices dated September 19, 2006 were issued informing the occupants thereon of the intended demolition of the erected structures on October 17 to 20, 2006. Consequently, Elvin Gumangan, Narciso Basatan and Lazaro Bawas (hereinafter private respondents) filed a petition for injunction with prayer for the issuance of a temporary restraining order and/or writ of preliminary injunction against the Office of the City Mayor of Baguio City through its Acting City Mayor, Reynaldo Bautista, the City Building and Architecture Office, the Anti-Squatting Task Force, and the Public Order and Safety Division, among others, (collectively called petitioners) before the National Commission on Indigenous Peoples, Cordillera Administrative Region (NCIP-CAR), Regional Hearing Office, La Trinidad, Benguet, docketed as Case No. 31-CAR-06.In their petition, private respondents basically claimed that the lands where their residential houses stand are their ancestral lands which they have been occupying and possessing openly and continuously since time immemorial; that their ownership thereof have been expressly recognized in Proclamation No. 15 dated April 27, 1922 and recommended by the Department of Environment and Natural Resources (DENR) for exclusion from the coverage of the Busol Forest Reserve. They, thus, contended that the demolition of their residential houses is a violation of their right of possession and ownership of ancestral lands accorded by the Constitution and the law, perforce, must be restrained. On October 16 and 19, 2006, Regional Hearing Officer Atty. Brain S. Masweng of the NCIP issued the two (2) assailed temporary restraining orders (TRO) directing the petitioners and all persons acting for and in their behalf to refrain from enforcing Demolition Advice dated September 18, 2006; Demolition Order dated September 19, 2006; Demolition Order No. 25, Series of 2004; Demolition Order No. 33, Series of 2005; and Demolition Order No. 28, Series of 2004, for a total period of twenty (20) days.Subsequently, the NCIP issued the other assailed Resolution dated November 10, 2006 granting the private respondents’ application for preliminary injunction subject to the posting of an injunctive bond each in the amount of P10,000.00.3Acting on the petition for certiorari filed by petitioners,4 the Court of Appeals upheld the jurisdiction of the NCIP over the action filed by private respondents and affirmed the temporary restraining orders dated October 165 and 19, 2006,6 and the Resolution dated November 10, 2006,7 granting the application for a writ of preliminary injunction, issued by the NCIP. The appellate court also ruled that Baguio City is not exempt from the coverage of Republic Act No. 8371, otherwise known as the Indigenous Peoples Rights Act of 1997 (IPRA).Petitioners assert that the NCIP has no jurisdiction to hear and decide main actions for injunction such as the one filed by private respondents. They claim that the NCIP has the authority to issue temporary restraining orders and writs of preliminary injunction only as auxiliary remedies to cases pending before it. Further, the IPRA provides that Baguio City shall be governed by its Charter. Thus, private respondents cannot claim their alleged ancestral lands under the provisions of the IPRA.Petitioners contend that private respondents are not entitled to the protection of an injunctive writ because they encroached upon the Busol Forest Reservation and built structures thereon without the requisite permit. Moreover, this Court, in Heirs of Gumangan v. Court of Appeals,8 had already declared that the Busol Forest Reservation is inalienable and

Page 2: pubcorp midterm digests.docx

possession thereof, no matter how long, cannot convert the same into private property. Even assuming that private respondents have a pending application for ancestral land claim, their right is at best contingent and cannot come under the protective mantle of injunction. Petitioners also claim that the Busol Forest Reservation is exempt from ancestral claims as it is needed for public welfare. It is allegedly one of the few remaining forests in Baguio City and is the city’s main watershed.Finally, petitioners contend that the demolition orders were issued pursuant to the police power of the local government.1avvphi1In their Comment9 dated March 1, 2007, private respondents defend the jurisdiction of the NCIP to take cognizance of and decide main actions for injunction arguing that the IPRA does not state that the NCIP may only issue such writs of injunction as auxiliary remedies. Private respondents also contend that the IPRA does not exempt Baguio City from its coverage nor does it state that there are no ancestral lands in Baguio City.As members of the Ibaloi Indigenous Community native to Baguio City, private respondents are treated as squatters despite the fact that they hold native title to their ancestral land. The IPRA allegedly now recognizes ancestral lands held by native title as never to have been public lands. Private respondents aver that the Busol Forest Reservation is subject to ancestral land claims. In fact, Proclamation No. 1510 dated April 27, 1922, which declared the area a forest reserve, allegedly did not nullify the vested rights of private respondents over their ancestral lands and even identified the claimants of the particular portions within the forest reserve. This claim of ownership is an exception to the government’s contention that the whole area is a forest reservation.Lastly, private respondents assert that the power of the city mayor to order the demolition of certain structures is not absolute. Regard should be taken of the fact that private respondents cannot be issued building permits precisely because they do not have paper titles over their ancestral lands, a requirement for the issuance of a building permit under the National Building Code.Petitioners’ Reply to Comment11 dated June 11, 2008 merely reiterates their previous arguments.We shall first dispose of the elemental issue of the NCIP’s jurisdiction. The NCIP is the primary government agency responsible for the formulation and implementation of policies, plans and programs to protect and promote the rights and well-being of indigenous cultural communities/indigenous peoples (ICCs/IPs) and the recognition of their ancestral domains as well as their rights thereto.12 In order to fully effectuate its mandate, the NCIP is vested with jurisdiction over all claims and disputes involving the rights of ICCs/IPs. The only condition precedent to the NCIP’s assumption of jurisdiction over such disputes is that the parties thereto shall have exhausted all remedies provided under their customary laws and have obtained a certification from the Council of Elders/Leaders who participated in the attempt to settle the dispute that the same has not been resolved.13In addition, NCIP Administrative Circular No. 1-03 dated April 9, 2003, known as the Rules on Pleadings, Practice and Procedure Before the NCIP, reiterates the jurisdiction of the NCIP over claims and disputes involving ancestral lands and enumerates the actions that may be brought before the commission. Sec. 5, Rule III thereof provides: Sec. 5. Jurisdiction of the NCIP.—The NCIP through its Regional Hearing Offices shall exercise jurisdiction over all claims and disputes involving rights of ICCs/IPs and all cases pertaining to the implementation, enforcement, and interpretation of R.A. 8371, including but not limited to the following:(1) Original and Exclusive Jurisdiction of the Regional Hearing Office (RHO):a. Cases involving disputes and controversies over ancestral lands/domains of ICCs/IPs;b. Cases involving violations of the requirement of free and prior and informed consent of ICCs/IPs;c. Actions for enforcement of decisions of ICCs/IPs involving violations of customary laws or desecration of ceremonial sites, sacred places, or rituals;d. Actions for redemption/reconveyance under Section 8(b) of R.A. 8371; ande. Such other cases analogous to the foregoing.(2) Original Jurisdiction of the Regional Hearing Officer:a. Cases affecting property rights, claims of ownership, hereditary succession, and settlement of land disputes, between and among ICCs/IPs that have not been settled under customary laws; andb. Actions for damages arising out of any violation of Republic Act No. 8371.(3) Exclusive and Original Jurisdiction of the Commission:a. Petition for cancellation of Certificate of Ancestral Domain Titles/Certificate of Ancestral Land Titles (CADTs/CALTs) alleged to have been fraudulently acquired by, and issued to, any person or community as provided for under Section 54 of R.A. 8371. Provided that such action is filed within one (1) year from the date of registration. In order to determine whether the NCIP has jurisdiction over the dispute in accordance with the foregoing provisions, it is necessary to resolve, on the basis of the allegations in their petition, whether private respondents are members of ICCs/IPs.

Page 3: pubcorp midterm digests.docx

In their petition14 filed before the NCIP, private respondents, members of the Ibaloi tribe who first settled in Baguio City, were asserting ownership of portions of the Busol Forest Reservation which they claim to be their ancestral lands. Correctly denominated as a petition for injunction as it sought to prevent the enforcement of the demolition orders issued by the City Mayor, the petition traced private respondents’ ancestry to Molintas and Gumangan and asserted their possession, occupation and utilization of their ancestral lands. The petition also alleged that private respondents’ claim over these lands had been recognized by Proclamation No. 15 which mentions the names of Molintas and Gumangan as having claims over portions of the Busol Forest Reservation.15 Clearly then, the allegations in the petition, which axiomatically determine the nature of the action and the jurisdiction of a particular tribunal,16 squarely qualify it as a "dispute(s) or controversy(s) over ancestral lands/domains of ICCs/IPs" within the original and exclusive jurisdiction of the NCIP-RHO.1avvphi1The IPRA, furthermore, endows the NCIP with the power to issue temporary restraining orders and writs of injunction. Sec. 69 thereof states:Sec. 69. Quasi-Judicial Powers of the NCIP.—The NCIP shall have the power and authority:a) To promulgate rules and regulations governing the hearing and disposition of cases filed before it as well as those pertaining to its internal functions and such rules and regulations as may be necessary to carry out the purposes of this Act;b) To administer oaths, summon the parties to a controversy, issue subpoenas requiring the attendance and testimony of witnesses or the production of such books, papers, contracts, records, agreements, and other document of similar nature as may be material to a just determination of the matter under investigation or hearing conducted in pursuance of this Act;c) To hold any person in contempt, directly or indirectly, and impose appropriate penalties therefor; andd) To enjoin any or all acts involving or arising from any case pending before it which, if not restrained forthwith, may cause grave or irreparable damage to any of the parties to the case or seriously affect social or economic activity. [Emphasis supplied]NCIP Administrative Circular No. 1-03 echoes the above-quoted provision in Sec. 82, Rule XV, which provides:Sec. 82. Preliminary Injunction and Temporary Restraining Order.—A writ of preliminary injunction or restraining order may be granted by the Commission pursuant to the provisions of Sections 59 and 69 of R.A. [No.] 8371 when it is established, on the basis of sworn allegations in a petition, that the acts complained of involving or arising from any case, if not restrained forthwith, may cause grave or irreparable damage or injury to any of the parties, or seriously affect social or economic activity. This power may also be exercised by RHOs in cases pending before them in order to preserve the rights of the parties.As can be gleaned from the foregoing provisions, the NCIP may issue temporary restraining orders and writs of injunction without any prohibition against the issuance of the writ when the main action is for injunction. The power to issue temporary restraining orders or writs of injunction allows parties to a dispute over which the NCIP has jurisdiction to seek relief against any action which may cause them grave or irreparable damage or injury. In this case, the Regional Hearing Officer issued the injunctive writ because its jurisdiction was called upon to protect and preserve the rights of private respondents who are undoubtedly members of ICCs/IPs.Parenthetically, in order to reinforce the powers of the NCIP, the IPRA even provides that no restraining order or preliminary injunction may be issued by any inferior court against the NCIP in any case, dispute or controversy arising from or necessary to the interpretation of the IPRA and other laws relating to ICCs/IPs and ancestral domains.17Petitioners argue that Baguio City is exempt from the provisions of the IPRA, and necessarily the jurisdiction of the NCIP, by virtue of Sec. 78 thereof, which states:SEC. 78. Special Provision.—The City of Baguio shall remain to be governed by its Charter and all lands proclaimed as part of its townsite reservation shall remain as such until otherwise reclassified by appropriate legislation: Provided, That prior land rights and titles recognized and/or acquired through any judicial, administrative or other processes before the effectivity of this Act shall remain valid: Provided, further, That this provision shall not apply to any territory which becomes part of the City of Baguio after the effectivity of this Act.lavvphil.net [Emphasis supplied]The foregoing provision indeed states that Baguio City is governed by its own charter. Its exemption from the IPRA, however, cannot ipso facto be deduced because the law concedes the validity of prior land rights recognized or acquired through any process before its effectivity. The IPRA demands that the city’s charter respect the validity of these recognized land rights and titles. The crucial question to be asked then is whether private respondents’ ancestral land claim was indeed recognized by Proclamation No. 15, in which case, their right thereto may be protected by an injunctive writ. After all, before a writ of preliminary injunction may be issued, petitioners must show that there exists a right to be protected and that the acts against which injunction is directed are violative of said right.18

Page 4: pubcorp midterm digests.docx

Proclamation No. 15, however, does not appear to be a definitive recognition of private respondents’ ancestral land claim. The proclamation merely identifies the Molintas and Gumangan families, the predecessors-in-interest of private respondents, as claimants of a portion of the Busol Forest Reservation but does not acknowledge vested rights over the same. In fact, Proclamation No. 15 explicitly withdraws the Busol Forest Reservation from sale or settlement. It provides:Pursuant to the provisions of section eighteen hundred and twenty-six of Act Numbered Twenty-seven Hundred and eleven[,] I hereby establish the Busol Forest Reservation to be administered by the Bureau of Forestry for the purpose of conserving and protecting water and timber, the protection of the water supply being of primary importance and all other uses of the forest are to be subordinated to that purpose. I therefore withdraw from sale or settlement the following described parcels of the public domain situated in the Township of La Trinidad, City of Baguio, Mountain Province, Island of Luzon, to wit:The fact remains, too, that the Busol Forest Reservation was declared by the Court as inalienable in Heirs of Gumangan v. Court of Appeals.19 The declaration of the Busol Forest Reservation as such precludes its conversion into private property. Relatedly, the courts are not endowed with jurisdictional competence to adjudicate forest lands. All told, although the NCIP has the authority to issue temporary restraining orders and writs of injunction, we are not convinced that private respondents are entitled to the relief granted by the Commission. WHEREFORE, the instant petition is GRANTED. The Decision of the Court of Appeals in CA G.R. SP No. 96895 dated April 16, 2007 and its Resolution dated September 11, 2007 are REVERSED and SET ASIDE. Case No. 31-CAR-06 entitled, Elvin Gumangan, Narciso Basatan and Lazaro Bawas v. Office of the City Mayor of Baguio City, et al. is DISMISSED. No pronouncement as to costs.Lagcao vs. LabraFacts: On July 9, 1986, the court a quo ruled in favor of petitioners and ordered the Province of Cebu to execute the final deed of sale in favor of petitioners. On June 11, 1992, the Court of Appeals affirmed the decision of the trial court. Pursuant to the ruling of the appellate court, the Province of Cebu executed on June 17, 1994 a deed of absolute sale over Lot 1029 in favor of petitioners. Thereafter, Transfer Certificate of Title (TCT) No. 129306 was issued in the name of petitioners and Crispina Lagcao.3After acquiring title, petitioners tried to take possession of the lot only to discover that it was already occupied by squatters. Thus, on June 15, 1997, petitioners instituted ejectment proceedings against the squatters. The Municipal Trial Court in Cities (MTCC), Branch 1, Cebu City, rendered a decision on April 1, 1998, ordering the squatters to vacate the lot. On appeal, the RTC affirmed the MTCC’s decision and issued a writ of execution and order of demolition.1avvphi1However, when the demolition order was about to be implemented, Cebu City Mayor Alvin Garcia wrote two letters4 to the MTCC, requesting the deferment of the demolition on the ground that the City was still looking for a relocation site for the squatters.Issue: Is Cebu City ordinance no. 1843 violative of substantive due processRuling: Yes, Ordinance No. 1843 to be constitutionally infirm for being violative of the petitioners’ right to due process.It should also be noted that, as early as 1998, petitioners had already obtained a favorable judgment of eviction against the illegal occupants of their property. The judgment in this ejectment case had, in fact, already attained finality, with a writ of execution and an order of demolition. But Mayor Garcia requested the trial court to suspend the demolition on the pretext that the City was still searching for a relocation site for the squatters. However, instead of looking for a relocation site during the suspension period, the city council suddenly enacted Ordinance No. 1843 for the expropriation of petitioners’ lot. It was trickery and bad faith, pure and simple. The unconscionable manner in which the questioned ordinance was passed clearly indicated that respondent City transgressed the Constitution, RA 7160 and RA 7279.

Antonio vs. Geronimo

The fundamental precept that underlies this case is that expropriation has no binding legal effect unless a formal expropriation proceeding has been instituted. A complaint for unlawful detainer[1] was filed before the Municipal Trial Court (MTC) of Antipolo,[2] docketed as Civil Case No. 2223 by Alexander Catolos (private respondent), who alleged that he was the registered owner of four (4) parcels of land[3] situated at Mayamot, Antipolo, Rizal, covered by Transfer Certificates of Title (TCT) Nos. 243003, 243179, 226192, and 166965, respectively. The defendants therein were the petitioners, who were occupying the said properties.

Page 5: pubcorp midterm digests.docx

Private respondent claimed he allowed petitioners to occupy portions of his land without requiring them to pay rent, on the condition that the latter would immediately vacate the same in the event that the former would need the premises. However, when private respondent did notify petitioners of his need to use the premises, petitioners refused to vacate the land even after demand. The complaint was resolved in favor of private respondent. In a Decision[4] dated 15 September 1993, respondent judge ordered petitioners to vacate the subject properties and pay the amount of Two Hundred Pesos (P200.00) as reasonable compensation for the use and occupation of the properties, as well as Twenty Thousand Pesos (P20,000.00) for litigation expenses and attorney’s fees. On 23 November 1993, private respondent filed a motion for issuance of a writ of demolition. The lower court granted the motion and directed the issuance of a writ of demolition. On 28 March 1994, a writ of demolition was issued. Partial demolition had already taken place by April 1994. Private respondent filed an urgent ex parte motion,[5] seeking the full implementation of the writ of demolition. This was granted on 24 April 1995, with respondent judge ordering the Chief of Police of the Regional Special Action Force in Canlubang, Laguna to provide assistance to Deputy Sheriff Eusebio J. Villaran in the implementation of the writ. On 20 June 1995, the Sangguniang Bayan of Antipolo, Rizal passed Resolution No. 61-95,[6] authorizing Mayor Daniel Garcia to acquire thru expropriation or purchase the subject properties for public purposes/socialized housing. Another resolution, No. 88-95, was issued on 30 August 1995 amending the previous resolution by further authorizing the municipal mayor to secure financing for the acquisition of the said parcel of land subject of this case. In Resolution No. 119-95[7] passed on 18 October 1995, the Sangguniang Bayan informed respondent court of the expropriation and the fact that the funds required for the same have already been included in the 1996 budget, and requested that the demolition be held in abeyance. At this point, the writ of demolition had not yet been fully implemented. The demolition proceeded despite said resolutions of the Sangguniang Bayan. On 28 November 1995, petitioners filed a motion to stay execution with the MTC on the ground that supervening events have rendered execution unjust and inequitable. A Supplement to the Motion to Stay Execution was filed on 1 March 1996. Petitioners invoked Commonwealth Act No. 538 in asking respondent judge to suspend the action for ejectment in view of the announced expropriation of the subject properties. The motion was denied in an Order[8] dated 29 March 1996. Respondent judge reasoned out that no action for expropriation had yet been filed in court and that petitioners had not complied with Commonwealth Act No. 538 in paying the current rents. On 2 May 1996, private respondent filed a motion for the issuance of alias writ of demolition. This was not resolved by the MTC. On the basis of this factual backdrop, petitioners filed the present petition for certiorari under Rule 65 on 17 May 1996. Petitioners seek to set aside the Order dated 29 March 1996 denying their motion to stay execution and to enjoin respondents from continuing with the demolition of their homes. They likewise pray for the issuance of writs of preliminary injunction and temporary restraining order. During the pendency of this case, private respondent filed a motion for re-issuance of a writ of demolition on 22 February 2001. In an Order dated 26 January 2004, the motion for re-issuance of writ of execution was granted. The MTC judge[9] issued an Alias Writ of Demolition in an Order dated 20 May 2004. Petitioners filed with the Court a Manifestation and Urgent Motion for Issuance of a TRO on 9 June 2004 to stop the enforcement of the said writ. Petitioners received a Notice to Vacate from the sheriff on 28 September 2004. On 11 October 2004, the residential houses and other structures owned by petitioners were demolished. This prompted

Page 6: pubcorp midterm digests.docx

petitioners to file an urgent request for early resolution of motion to issue temporary restraining order and an urgent manifestation with urgent motion to resolve motion for issuance of a status quo order on 20 October 2004. Petitioners ascribe grave abuse of discretion amounting to lack of jurisdiction to respondent judge in denying the motion to stay execution in view of two considerations, namely: the supervening events which make execution unjust and inequitable, and non-compliance with Commonwealth Act No. 538[10]. Petitioners argue that Commonwealth Act No. 538 clearly provides that for the purposes of the Act, the action shall be considered instituted from the time the competent authority advises in writing the owner of the intention of the government to acquire his land. In its Comment,[11] private respondent posits that Commonwealth Act No. 538 applies only to cases wherein expropriation proceedings are filed. The Sangguniang Bayan’s resolutions expressing the intent to expropriate the properties evinced merely an intention to expropriate and not the actual expropriation proceeding. According to private respondent, assuming that there exists an expropriation proceeding, petitioners still cannot avail of the automatic suspension of the ejectment case because they failed to pay their current rentals and deposit them with the court. Regarding the issue of non-payment of rentals, petitioners point out that facts of record show that they were not required by contract to pay rents, thereby rendering the requirement cited by private respondent inapplicable. The issues raised by petitioners can be synthesized into one: Whether a resolution for expropriation by a local government unit can suspend the writ of execution and demolition in an ejectment case. In actions for ejectment, the general rule is if judgment is rendered against the defendant, it is immediately executory. Such judgment, however, may be stayed by the defendant only by: (a) perfecting an appeal; (b) filing a supersedeas bond; and (c) making a periodic deposit of the rental or the reasonable compensation for the use and occupation of the property during the pendency of the appeal.[12] These requisites must concur. Thus, even if the defendant had appealed and filed a supersedeas bond but failed to pay the accruing rentals, the appellate court could, upon motion of the plaintiff with notice to the defendant, and upon proof of such failure, order the immediate execution of the appealed decision without prejudice to the appeal taking its course. Such deposit, like the supersedeas bond, is a mandatory requirement; hence, if it is not complied with, execution will issue as a matter of right.[13] Jurisprudence is replete with cases which provide for the exceptions to the rule cited above. These are the existence of fraud, accident, mistake or excusable negligence which prevented the defendant from making the monthly deposit, or the occurrence of supervening events which have brought about a material change in the situation of the parties and would make the execution inequitable or where there is compelling urgency for the execution because it is not justified by the prevailing circumstances.[14] Petitioners anchor their argument to stay execution upon this supervening event in the form of a resolution for expropriation filed by the Sangguniang Bayan. Petitioners claim that this logically begs the question of whether this act of the local legislative body of Antipolo decreeing expropriation of the property subject of the ejectment case constitutes a supervening event which makes execution of the ejectment decision unjust and impractical. The power of eminent domain, inherent in the State, employs private property for public use upon payment of just compensation. The enactment of the principle of social justice falls within the parameters of public use. In the case of Marina Reyes v. National Housing Authority,[15] this Court, speaking thru Justice Reynato Puno, stated: Moreover the Constitution itself allows the State to undertake, for the common good and in cooperation with the private sector, a continuing program of urban land reform and housing which will make at affordable cost decent housing and basic services to underprivileged and homeless citizens in urban centers and resettlement areas. The expropriation of private property for the purpose of socialized housing for the marginalized sector is in furtherance of the social justice provision under Section 1, Article XIII of the Constitution which provides that: SECTION 1. The Congress shall give highest priority to the enactment of measures that protect and enhance the right of all the people to human dignity, reduce social, economic, and political inequalities, and remove cultural inequities by equitably diffusing wealth and political power for the common good.

Page 7: pubcorp midterm digests.docx

To this end, the State shall require the acquisition, ownership, use and disposition of property and its increments. Local government units may exercise the power of eminent domain, subject to the limitation embodied under the law. There are two relevant laws to consider, the Local Government Code (LGC) and Republic Act No. 7279 (UDHA). We first look at Section 19 of the LGC which establishes how a local government unit may expropriate private property, thus: SEC. 19. Eminent Domain. — A local government unit may, through its chief executive and acting pursuant to an ordinance, exercise the power of eminent domain for public use, or purpose, or welfare for the benefit of the poor and the landless, upon payment of just compensation, pursuant to the provisions of the Constitution and pertinent laws: Provided, however, That the power of eminent domain may not be exercised unless a valid and definite offer has been previously made to the owner, and such offer was not accepted: Provided, further, That the local government unit may immediately take possession of the property upon the filing of the expropriation proceedings and upon making a deposit with the proper court of at least fifteen percent (15%) of the fair market value of the property based on the current tax declaration of the property to be expropriated: Provided, finally, That, the amount to be paid for the expropriated property shall be determined by the proper court, based on the fair market value at the time of the taking of the property. Clearly, the Sangguniang Bayan, being a local legislative body, may exercise the power to expropriate private properties, subject to the following requisites, all of which must concur: 1. An ordinance is enacted by the local legislative council authorizing the local chief executive, in behalf of the local government unit, to exercise the power of eminent domain or pursue expropriation proceedings over a particular private property.2. The power of eminent domain is exercised for public use, purpose or welfare, or for the benefit of the poor and the landless.3. There is payment of just compensation, as required under Section 9, Article III of the Constitution, and other pertinent laws.4. A valid and definite offer has been previously made to the owner of the property sought to be expropriated, but said offer was not accepted.[16] In the instant case, no ordinance was passed by the Sangguniang Bayan of Antipolo. In its stead were Resolution Nos. 61-95 and 88-95. It has been categorically stated in the cases of Municipality of Paranaque v. V.M. Realty Corporation[17] and Heirs of Suguitan v. City of Mandaluyong[18] that a local government unit cannot authorize an expropriation of private property through a mere resolution of its lawmaking body, thus: We are not convinced by petitioner's insistence that the terms "resolution" and "ordinance" are synonymous. A municipal ordinance is different from a resolution. An ordinance is a law, but a resolution is merely a declaration of the sentiment or opinion of a lawmaking body on a specific matter. An ordinance possesses a general and permanent character, but a resolution is temporary in nature. Additionally, the two are enacted differently — a third reading is necessary for an ordinance, but not for a resolution, unless decided otherwise by a majority of all the Sanggunian members. If Congress intended to allow LGUs to exercise eminent domain through a mere resolution, it would have simply adopted the language of the previous Local Government Code. But Congress did not. In a clear divergence from the previous Local Government Code, Section 19 of RA 7160 categorically requires that the local chief executive act pursuant to an ordinance. . . . Moreover, the power of eminent domain necessarily involves a derogation of a fundamental or private right of the people. Accordingly, the manifest change in the legislative language — from "resolution" under the BP 337 to "ordinance" under RA 7160 — demands a strict construction. "No species of property is held by individuals with greater tenacity, and is guarded by the Constitution and laws more sedulously, than the right to the freehold of inhabitants. When the legislature interferes with that right and, for greater public purposes, appropriates the land of an individual without his consent, the plain meaning of the law should not be enlarged by doubtful interpretation." [19] (Emphasis supplied.)

Page 8: pubcorp midterm digests.docx

These resolutions cannot partake of a supervening event so as to suspend the writ of execution in the ejectment proceedings. They merely express at most an intention to expropriate. Private respondent correctly maintained that there was no positive act of instituting the intended expropriation proceedings.[20] Assuming arguendo that instead of resolutions, an ordinance was passed by the Sangguniang Bayan, we still find for private respondent. There is no dispute that a local government unit possesses the power of eminent domain. But the taking of private properties is not absolute. The power of eminent domain must not be exercised arbitrarily, even if purposed for resolving a critical problem such as urban squatting. The safeguards afforded by law require strict observance. In Filstream International Incorporated v. Court of Appeals,[21] we state: We take judicial notice of the fact that urban land reform has become a paramount task in view of the acute shortage of decent housing in urban areas particularly in Metro Manila. Nevertheless, despite the existence of a serious dilemma, local government units are not given an unbridled authority when exercising their power of eminent domain in pursuit of solutions to these problems. The basic rules still have to be followed, which are as follows: "no person shall be deprived of life, liberty, or property without due process of law, nor shall any person be denied the equal protection of the laws (Art. 3, Sec. 1, 1987 Constitution); private property shall not be taken for public use without just compensation (Art. 3, Section 9, 1987 Constitution)." The UDHA is also relevant as it governs the local expropriation of property for purposes of urban land reform and housing. Sections 9 and 10, in particular, provide the relevant limitations, thus: Sec. 9. Priorities in the Acquisition of Land.— Lands for socialized housing shall be acquired in the following order: (a) Those owned by the government or any of its subdivisions, instrumentalities, or agencies, including government-owned or -controlled corporations and their subsidiaries; (b) Alienable lands of the public domain; (c) Unregistered or abandoned and idle lands; (d) Those within the declared Areas for Priority Development, Zonal Improvement Program sites, and Slum Improvement and Resettlement Program sites which have not yet been acquired; (e) Bagong Lipunan Improvement of Sites and Services or BLISS sites which have not yet been acquired; and (f) Privately-owned lands. Where open-site development is found more practicable and advantageous to the beneficiaries, the priorities mentioned in this section shall not apply. The local government units shall give budgetary priority to on-site development of government lands. Sec. 10. Modes of Land Acquisition.—The modes of acquiring lands for purposes of this Act shall include, among others, community mortgage, land swapping, land assembly or consolidation, land banking, donation to the government, joint-venture agreement, negotiated purchase, and expropriation: Provided, however, That expropriation shall be resorted to only when other models of acquisition have been exhausted: Provided, further, That where expropriation is resorted to, parcels of land owned by small property owners shall be exempted for purposes of this Act: Provided, finally, That abandoned property, as herein defined, shall be reverted and escheated to the State in a proceeding analogous to the procedure laid down in Rule 91 of the Rules of Court. For the purpose of socialized housing, government-owned and foreclosed properties shall be acquired by the local government units, or by the National Housing Authority primary through negotiated purchase: Provided, That qualified beneficiaries who are actual occupants of the land shall be given the right of first refusal. Assuming that there was even an attempt by Antipolo City to expropriate the subject properties for socialized housing, the records do not show compliance with the abovementioned rules. No attempt was made to acquire the first five (5) lands mentioned in Section 9. Neither were the other modes of acquisition exhausted, as mandated by Section 10. An examination of the resolutions readily shows that the purpose for which they were passed is to save petitioners from the

Page 9: pubcorp midterm digests.docx

impending demolition.[22] Yet nothing therein is of binding force as to preclude the ejectment of the petitioners or the demolition of their houses. In its second assignment of error, petitioners fault respondent judge for holding their non-compliance with Commonwealth Act No. 538. Petitioners aver that the law does not require that an action for expropriation be filed in court before the suspension of the action for ejectment shall be automatically suspended.[23] Moreover, they assert that the requirement on the payment of rents as a precondition to the availment of the benefits of suspension does not apply to them because they were allowed by private respondent to occupy the premises without paying rent. Commonwealth Act No. 538[24] took effect on 26 May 1940. Republic Act No. 1162 was enacted on 18 June 1954. Section 5[25] of said Act provides that from the approval of said Act and until the expropriation herein provided, no ejectment proceedings shall be instituted or prosecuted against any tenant or occupant if he pays his current rentals. The interpretation of this provision, as placed by the Court in numerous cases,[26] is that an action for ejectment is deemed suspended only when an expropriation proceeding is actually commenced. Apparently to cure the objection that by the indefinite period of suspension provided for in section 5 of Republic Act No. 1162 the owner would be deprived indefinitely of his property without due process of law, Republic Act No. 1599 was enacted amending, among others, that section by expressly providing that the suspension of ejectment proceedings shall be limited to the period of two years from the date of the approval of the amendatory act, namely, June 17, 1956.[27] In Teresa Realty v. Potenciano,[28] the Court was emphatic in stating that the remedies granted to tenants under section 5, Republic Act No. 1162 or its amendment, Republic Act No. 1599, whether by way of suspension of ejectment actions, payment of rent by installments, or rental ceilings, do not and can not apply where expropriation proceedings have not been commenced under pain of unconstitutionality. On 3 August 1959, Congress approved another similar law, Republic Act No. 2616, Section 4 of which provides: SEC. 4. After the expropriation proceedings mentioned in section two of this Act shall have been initiated and during the pendency of the same, no ejectment proceedings shall be instituted or prosecuted against the present occupant of any lot in said Tatalon Estate, and no ejectment proceedings already commenced shall be continued, and such lot or any portion thereof shall not be sold by the owners of said estate to any person other than the present occupant without the consent of the latter given in a public instrument. Although the said provision survived the challenge of being discriminatory, the Court, inferring from the twin cases of J.M. Tuason & Co., Inc. v. Court of Appeals, et al. and Republic of the Philippines v. J.M. Tuason & Co., Inc., et. al.,[29] imposed guidelines for its implementation, such that an ejectment proceeding cannot be barred or suspended under Republic Act No. 2616 unless an action for expropriation is actually filed; the government takes possession of the land; and coetaneous payment of just compensation is made.[30] The Court explained in Familara v. J.M. Tuason,[31] thus: Definitely, to hold that the mere declaration of an intention to expropriate, without instituting the corresponding proceeding therefor before the courts, with assurance of just compensation, would already preclude the exercise by the owner of his rights of ownership over the land, or bar the enforcement of any final ejectment order that the owner may have obtained against any intruder into the land, is to sanction an act which is indeed confiscatory and therefore offensive to the Constitution. For it must be realized that in a condemnation case, it is from the condemnor's taking possession of the property that the owner is deprived of the benefits of ownership, such as possession, management and disposition thereof. Before that time, the proprietary right of the owner over his property must be recognized. On June 16, 1962, Republic Act No. 3453 amended Section 4 of Republic Act No. 2616 as follows: SEC. 4. Upon approval of this amendatory Act, no ejectment proceedings shall be instituted or prosecuted against the present occupants of any lot in said Tatalon Estate, and no ejectment proceedings already commenced shall be continued, and such lot or any portion thereof shall not be sold by the owners of said estate to any person other than the present occupant without the consent of the latter given in a public instrument.

Page 10: pubcorp midterm digests.docx

In Cuatico v. Court of Appeals,[32] the Court struck down Republic Act No. 3453 for being confiscatory because it allows the continuance of the occupation of the land on the part of the tenant indefinitely even if no expropriation proceedings are taken or contemplated, thus taking from the owner his property without compensation and depriving him of his dominical rights of ownership over it without due process in violation of the Constitution. Taking a cue from the rationale behind the unenforceability of these provisions on the suspension of ejectment proceedings, we agree with private respondent that Commonwealth Act No. 538 applies only to cases where there exist actual expropriation proceedings. Considering petitioners’ admission that they do not pay rentals, it bears noting that in Javier v. Araneta[33] therein petitioner was not allowed to invoke the benefits of Section 1 of Commonwealth Act No. 538. We quote: In the second place, the law provides that “to avail himself of the benefits of the suspension, the tenant shall pay the land owner the current rents as they become due or deposit the same with the court where the action for ejectment has been instituted. . . .” Petitioner herein does not come with the purview of this provision. She is a mere squatter and, therefore, not a tenant within a legal meaning of this word. The word “tenant” does not include a squatter, because a squatter is “a person who settles or locates on land, inclosed or uninclosed, with no bona fide claim or color of title, and without the consent of the owner; and it does not appear that petitioner has paid to the land owner the current rents as they became due or has deposited the same with the court where the action for ejectment has been instituted. Petitioner, therefore, can not invoke the benefits of said Commonwealth Act No. 538. We reiterate now, as then, that only tenants who have been in faithful payment of rentals may invoke the benefits under Commonwealth Act No. 538. Admittedly, the ejectment of petitioners and the demolition of their houses does nothing to resolve the crippling problem of urban tenancy. Yet our concern is the enforcement of legal rights, and petitioners’ right to occupy the property of another is deficient under the law. Certainly, the government of Antipolo City was well capacitated to enact a valid measure of legal protection to petitioners, yet the manner by which they attempted to do so was ineffectual. WHEREFORE, the petition is DISMISSED.

MMDA vs. Garin

Facts: The issue arose from an incident involving the respondent Dante O. Garin, a lawyer, who was issued a traffic violation receipt (TVR) and his driver's license confiscated for parking illegally along Gandara Street, Binondo, Manila, on 05 August 1995. The following statements were printed on the TVR:You are hereby directed to report to the MMDA Traffic Operations Center Port Area Manila after 48 hours from date of apprehension for disposition/appropriate action thereon. Criminal case shall be filed for failure to redeem license after 30 days.Valid as temporary DRIVER'S license for seven days from date of apprehension.1Shortly before the expiration of the TVR's validity, the respondent addressed a letter2 to then MMDA Chairman Prospero Oreta requesting the return of his driver's license, and expressing his preference for his case to be filed in court.Issue: Did Rep. Act no. 7924 vest MMDA police power/Ruling: No, Rep. Act No. 7924 does not grant the MMDA with police power, let alone legislative power, and that all its functions are administrative in nature.Tracing the legislative history of Rep. Act No. 7924 creating the MMDA, we concluded that the MMDA is not a local government unit or a public corporation endowed with legislative power, and, unlike its predecessor, the Metro Manila Commission, it has no power to enact ordinances for the welfare of the community. Thus, in the absence of an ordinance from the City of Makati, its own order to open the street was invalid.

MMDA vs. Vron Transportation

Page 11: pubcorp midterm digests.docx

Facts: Pursuant to the E.O., the Metro Manila Council (MMC), the governing board and policymaking body of the MMDA, issued Resolution No. 03-07 series of 20037 expressing full support of the Project. Recognizing the imperative to integrate the different transport modes via the establishment of common bus parking terminal areas, the MMC cited the need to remove the bus terminals located along major thoroughfares of Metro Manila.8On February 24, 2003, Viron Transport Co., Inc. (Viron), a domestic corporation engaged in the business of public transportation with a provincial bus operation,9 filed a petition for declaratory relief10 before the RTC11 of Manila.In its petition which was docketed as Civil Case No. 03-105850, Viron alleged that the MMDA, through Chairman Fernando, was "poised to issue a Circular, Memorandum or Order closing, or tantamount to closing, all provincial bus terminals along EDSA and in the whole of the Metropolis under the pretext of traffic regulation."12 This impending move, it stressed, would mean the closure of its bus terminal in Sampaloc, Manila and two others in Quezon City.

Issue: Will the elimination of the terminal a valid exercise of police powers by the MMDARuling: No, the MMDA cannot order the closure of respondents’ terminals not only because no authority to implement the Project has been granted nor legislative or police power been delegated to it, but also because the elimination of the terminals does not satisfy the standards of a valid police power measure.

Lucena Grand Terminal vs. JAC Liner Facts: Respondent, JAC Liner, Inc., a common carrier operating buses which ply various routes to and from Lucena City, assailed, via a petition for prohibition and injunction[1] against the City of Lucena, its Mayor, and the Sangguniang Panlungsod of Lucena before the Regional Trial Court (RTC) of Lucena City, City Ordinance Nos. 1631 and 1778 as unconstitutional on the ground that, inter alia, the same constituted an invalid exercise of police power, an undue taking of private property, and a violation of the constitutional prohibition against monopoliesIssue: Is Lucena Ordinance no. 1631 a valid exercise of police power?Ruling:Yes, City Ordinance No. 1631 as valid, having been issued in the exercise of the police power of the City Government of Lucena insofar as the grant of franchise to the Lucena Grand Central Terminal, Inc., to construct, finance, establish, operate and maintain common bus-jeepney terminal facility in the City of Lucena

OSG vs. Ayala Land Inc.Facts: Before this Court is a Petition for Review on Certiorari, under Rule 45 of the Revised Rules of Court, filed by petitioner Office of the Solicitor General (OSG), seeking the reversal and setting aside of the Decision dated 25 January 2007 of the Court of Appeals in CA-G.R. CV No. 76298, which affirmed in toto the Joint Decision dated 29 May 2002 of the Regional Trial Court (RTC) of Makati City, Branch 138, in Civil Cases No. 00-1208 and No. 00-1210; and (2) the Resolution dated 14 March 2007 of the appellate court in the same case which denied the Motion for Reconsideration of the OSG. The RTC adjudged that respondents Ayala Land Incorporated (Ayala Land), Robinsons Land Corporation (Robinsons), Shangri-la Plaza Corporation (Shangri-la), and SM Prime Holdings, Inc. (SM Prime) could not be obliged to provide free parking spaces in their malls to their patrons and the general public.Issue: Is requiring owners to provide free parking, valid exercise of police power?Ruling: No, in totally prohibiting respondents from collecting parking fees from the public for the use of the mall parking facilities, the State would be acting beyond the bounds of police power. Police power is the power of promoting the public welfare by restraining and regulating the use of liberty and property. It is usually exerted in order to merely regulate the use and enjoyment of the property of the owner. The power to regulate, however, does not include the power to prohibit. A fortiori, the power to regulate does not include the power to confiscate.

Corona vs. United Harbor Pilots Assn. of the Phils.Facts: the PPA was created on July 11, 1974, by virtue of Presidential Decree No. 505. On December 23, 1975, Presidential Decree No. 857 was issued revising the PPA’s charter. Pursuant to its power of control, regulation, and supervision of pilots and the pilotage profession, the PPA promulgated PPA-AO-03-85 on March 21, 1985, which embodied the “Rules and Regulations Governing Pilotage Services, the Conduct of Pilots and Pilotage Fees in Philippine Ports.” These rules mandate, inter alia, that aspiring pilots must be holders of pilot licenses and must train as probationary pilots in outports for three

Page 12: pubcorp midterm digests.docx

months and in the Port of Manila for four months. It is only after they have achieved satisfactory performance that they are given permanent and regular appointments by the PPA itselfIssue: Is pilotage a property right? Is A.O. 04-92 violative of due process?Ruling: Yes, The court a quo pointed out that the Bureau of Customs, the precursor of the PPA, recognized pilotage as a profession and, therefore, a property rightNo, PPA-AO 04-92 does not constitute a wrongful interference with, let alone a wrongful deprivation of, the property rights of those affected thereby. As may be noted, the issuance aims no more than to improve pilotage services by limiting the appointment to harbor pilot positions to one year, subject to renewal or cancellation after a rigid evaluation of the appointee’s performance. PPA-AO 04-92 does not forbid, but merely regulates, the exercise by harbor pilots of their profession in PPA’s jurisdictional area

___________________________________________________________________________________________________________

Canlas vs. Napico Homeowners Assn. Facts: Petitioners were deprived of their liberty, freedom and/or rights to shelter enshrined and embodied in our Constitution, as the result of these nefarious activities of both the Private and Public Respondents. This ardent request filed before this Honorable Supreme Court is the only solution to this problem via this newly advocated principles incorporated in the Rules - the "RULE ON THE WRIT OF AMPAROIssue: Is a Threatened demolition of a dwelling within the right of life, liberty and property?Ruling: No, The threatened demolition of a dwelling by virtue of a final judgment of the court, which in this case was affirmed with finality by this Court, is not included among the enumeration of rights as stated in the above-quoted Section 1 for which the remedy of a writ of amparo is made available. Their claim to their dwelling, assuming they still have any despite the final and executory judgment adverse to them, does not constitute right to life, liberty and security. There is, therefore, no legal basis for the issuance of the writ of amparo.

G.R. No. 153974 August 7, 2006MIGUEL BELUSO, NATIVIDAD BELUSO, PEDRO BELUSO, ANGELITA BELUSO, RAMON BELUSO, and AMADA DANIEL, substituted by her heirs represented by TERESITA ARROBANG, Petitioners,vs.THE MUNICIPALITY OF PANAY (CAPIZ), represented by its Mayor, VICENTE B. BERMEJO, Respondent.D E C I S I O NAUSTRIA-MARTINEZ, J.:Before this Court is a petition for review questioning the Decision 1 of the Court of Appeals (CA) dated March 20, 2002 in CA-G.R. SP No. 47052, as well the Resolution 2 dated June 11, 2002 denying petitioners’ Motion for Reconsideration thereof. The facts are as follows:Petitioners are owners of parcels of land with a total area of about 20,424 square meters, covered by Free Patent Nos. 7265, 7266, 7267, 7268, 7269, and 7270. 3 On November 8, 1995, the Sangguniang Bayan of the Municipality of Panay issued Resolution No. 95-29 authorizing the municipal government through the mayor to initiate expropriation proceedings. 4 A petition for expropriation was thereafter filed on April 14, 1997 by the Municipality of Panay (respondent) before the Regional Trial Court (RTC), Branch 18 of Roxas City, docketed as Civil Case No. V-6958. 5Petitioners filed a Motion to Dismiss alleging that the taking is not for public use but only for the benefit of certain individuals; that it is politically motivated because petitioners voted against the incumbent mayor and vice-mayor; and that some of the supposed beneficiaries of the land sought to be expropriated have not actually signed a petition asking for the property but their signatures were forged or they were misled into signing the same. 6 On July 31, 1997, the trial court denied petitioners’ Motion to Dismiss and declared that the expropriation in this case is for "public use" and the respondent has the lawful right to take the property upon payment of just compensation. 7 Petitioners filed an Answer on August 12, 1997 reasserting the issues they raised in their Motion to Dismiss. 8 On October 1, 1997, the trial court issued an Order appointing three persons as Commissioners to ascertain the amount of just compensation for the property. 9 Petitioners filed a "Motion to Hold in Abeyance the Hearing of the Court Appointed Commissioners to Determine Just Compensation and for Clarification of the Court’s Order dated October 1, 1997" which

Page 13: pubcorp midterm digests.docx

was denied by the trial court on November 3, 1997. 10 Petitioners’ Motion for Reconsideration was also denied on December 9, 1997. 11 Petitioners then filed on March 2, 1998 a Petition for Certiorari before the CA claiming that they were denied due process when the trial court declared that the taking was for public purpose without receiving evidence on petitioners’ claim that the Mayor of Panay was motivated by politics in expropriating their property and in denying their Motion to Hold in Abeyance the Hearing of the Court Appointed Commissioners; and that the trial court also committed grave abuse of discretion when it disregarded the affidavits of persons denying that they signed a petition addressed to the municipal government of Panay. 12 On January 17, 2001, petitioners filed a Motion to Admit Attached Memorandum and the Memorandum itself where they argued that based on the Petition for Expropriation filed by respondent, such expropriation was based only on a resolution and not on an ordinance contrary to Sec. 19 of Republic Act (R.A.) No. 7160; there was also no valid and definite offer to buy the property as the price offered by respondent to the petitioners was very low. 13On March 20, 2002, the CA rendered its Decision dismissing the Petition for Certiorari. It held that the petitioners were not denied due process as they were able to file an answer to the complaint and were able to adduce their defenses therein; and that the purpose of the taking in this case constitutes "public use". 14 Petitioners filed a Motion for Reconsideration which was denied on June 11, 2002. 15 Thus, the present petition claiming that:A. RESPONDENT IS WITHOUT, LACKS AND DOES NOT HAVE THE LAWFUL POWER TO ACQUIRE ANY OR ALL OF THE SUBJECT PROPERTIES THROUGH EMINENT DOMAIN, IT BEING EXERCISED BY MEANS OF A MERE RESOLUTION, AND NOT THROUGH AN ORDINANCE AS REQUIRED BY LAW AND APPLICABLE JURISPRUDENCE;B. RESPONDENT IS LIKEWISE WITHOUT, LACKS AND DOES NOT HAVE THE LAWFUL POWER TO ACQUIRE ANY OR ALL OF THE SUBJECT PROPERTIES THROUGH EMINENT DOMAIN, ITS PREVIOUS OFFER TO BUY THEM BEING NOT VALID; andC. IT WAS A SERIOUS ERROR ON THE PART OF THE HONORABLE COURT OF APPEALS NOT TO DISCUSS, MUCH LESS RULE ON, BOTH IN ITS QUESTIONED DECISION AND ITS RESOLUTION PROMULGATED ON 11 JUNE 2002 PETITIONERS’ ARGUMENTS THAT RESPONDENT IS WITHOUT, LACKS AND DOES NOT HAVE THE LAWFUL POWER TO ACQUIRE ANY OR ALL OF THE SUBJECT PROPERTIES THROUGH EMINENT DOMAIN, IT BEING EXERCISED BY MEANS OF A MERE RESOLUTION, AND NOT THROUGH AN ORDINANCE AS REQUIRED BY LAW AND APPLICABLE JURISPRUDENCE, AND ITS PREVIOUS OFFER TO BUY THEM BEING NOT VALID, DESPITE THE FACT THAT THESE OBJECTIONS WERE PROPERLY PLEADED IN PETITIONERS’ MEMORANDUM WHICH WAS DULY ADMITTED IN ITS RESOLUTION PROMULGATED ON 29 JANUARY 2001; andD. PETITIONERS WERE UTTERLY DENIED PROCEDURAL DUE PROCESS OF LAW BY THE COURT A QUO, WHEN IT SIMPLY DECLARED IN ITS ORDER DATED 31 JULY 1997 THAT THE TAKING BY RESPONDENT OF PETITIONERS’ PROPERTIES IS PURPORTEDLY FOR PUBLIC PURPOSE WITHOUT RECEIVING EVIDENCE ON THEIR ASSERTED CLAIM THAT RESPONDENT’S MUNICIPAL MAYOR WAS POLITICALLY MOTIVATED IN SEEKING THE EXPROPRIATION OF THEIR PROPERTIES AND NOT FOR PUBLIC PURPOSE. 16 Petitioners argue that: contrary to Sec. 19 of R.A. No. 7160 of the Local Government Code, which provides that a local government may exercise the power of eminent domain only by "ordinance," respondent’s expropriation in this case is based merely on a "resolution"; while objection on this ground was neither raised by petitioners in their Motion to Dismiss nor in their Answer, such objection may still be considered by this Court since the fact upon which it is based is apparent from the petition for expropriation itself; a defense may be favorably considered even if not raised in an appropriate pleading so long as the facts upon which it is based are undisputed; courts have also adopted a more censorious attitude in resolving questions involving the proper exercise of local bodies of the delegated power of expropriation, as compared to instances when it is directly exercised by the national legislature; respondent failed to give, prior to the petition for expropriation, a previous valid and definite offer to petitioners as the amount offered in this case was only P10.00 per square meter, when the properties are residential in nature and command a much higher price; the CA failed to discuss and rule upon the arguments raised by petitioners in their Memorandum; attached to the Motion to Dismiss were affidavits and death certificates showing that there were people whose names were in the supposed petition asking respondent for land, but who did not actually sign the same, thus showing that the present expropriation was not for a public purpose but was merely politically motivated; considering the conflicting claims regarding the purpose for which the properties are being expropriated and inasmuch as said issue may not be rightfully ruled upon merely on the basis of petitioners’ Motion to Dismiss and Answer as well as respondent’s Petition for Expropriation, what should have been done was for the RTC to conduct hearing where each party is given ample opportunity to prove its claim. 17 Respondent for its part contends that its power to acquire private property for public use upon payment of just compensation was correctly upheld by the trial court; that the CA was correct in finding that the petitioners were not denied due process, even though no hearing was conducted in the trial court, as petitioners were still able to adduce their

Page 14: pubcorp midterm digests.docx

objections and defenses therein; and that petitioners’ arguments have been passed upon by both the trial court and the CA and were all denied for lack of substantial merit. 18Respondent filed a Memorandum quoting at length the decision of the CA to support its position. 19 Petitioners meanwhile opted to have the case resolved based on the pleadings already filed. 20We find the petition to be impressed with merit.Eminent domain, which is the power of a sovereign state to appropriate private property to particular uses to promote public welfare, is essentially lodged in the legislature. 21 While such power may be validly delegated to local government units (LGUs), other public entities and public utilities the exercise of such power by the delegated entities is not absolute. 22 In fact, the scope of delegated legislative power is narrower than that of the delegating authority and such entities may exercise the power to expropriate private property only when authorized by Congress and subject to its control and restraints imposed through the law conferring the power or in other legislations. 23 Indeed, LGUs by themselves have no inherent power of eminent domain. 24 Thus, strictly speaking, the power of eminent domain delegated to an LGU is in reality not eminent but "inferior" since it must conform to the limits imposed by the delegation and thus partakes only of a share in eminent domain. 25 The national legislature is still the principal of the LGUs and the latter cannot go against the principal’s will or modify the same. 26 The exercise of the power of eminent domain necessarily involves a derogation of a fundamental right. 27 It greatly affects a landowner’s right to private property which is a constitutionally protected right necessary for the preservation and enhancement of personal dignity and is intimately connected with the rights to life and liberty. 28 Thus, whether such power is exercised directly by the State or by its authorized agents, the exercise of such power must undergo painstaking scrutiny. 29 Indeed, despite the existence of legislative grant in favor of local governments, it is still the duty of the courts to determine whether the power of eminent domain is being exercised in accordance with the delegating law. Sec. 19 of R.A. No. 7160, which delegates to LGUs the power of eminent domain expressly provides:SEC. 19. Eminent Domain. - A local government unit may, through its chief executive and acting pursuant to an ordinance, exercise the power of eminent domain for public use, or purpose, or welfare for the benefit of the poor and the landless, upon payment of just compensation, pursuant to the provisions of the Constitution and pertinent laws: Provided, however, That the power of eminent domain may not be exercised unless a valid and definite offer has been previously made to the owner, and such offer was not accepted: Provided, further, That the local government unit may immediately take possession of the property upon the filing of the expropriation proceedings and upon making a deposit with the proper court of at least fifteen percent (15%) of the fair market value of the property based on the current tax declaration of the property to be expropriated: Provided, finally, That, the amount to be paid for the expropriated property shall be determined by the proper court, based on the fair market value at the time of the taking of the property. It is clear therefore that several requisites must concur before an LGU can exercise the power of eminent domain, to wit:1. An ordinance is enacted by the local legislative council authorizing the local chief executive, in behalf of the local government unit, to exercise the power of eminent domain or pursue expropriation proceedings over a particular private property.2. The power of eminent domain is exercised for public use, purpose or welfare, or for the benefit of the poor and the landless.3. There is payment of just compensation, as required under Section 9, Article III of the Constitution, and other pertinent laws.4. A valid and definite offer has been previously made to the owner of the property sought to be expropriated, but said offer was not accepted. 30 The Court in no uncertain terms have pronounced that a local government unit cannot authorize an expropriation of private property through a mere resolution of its lawmaking body. 31 R.A. No. 7160 otherwise known as the Local Government Code expressly requires an ordinance for the purpose and a resolution that merely expresses the sentiment of the municipal council will not suffice. 32A resolution will not suffice for an LGU to be able to expropriate private property; and the reason for this is settled:x x x A municipal ordinance is different from a resolution. An ordinance is a law, but a resolution is merely a declaration of the sentiment or opinion of a lawmaking body on a specific matter. An ordinance possesses a general and permanent character, but a resolution is temporary in nature. Additionally, the two are enacted differently -- a third reading is necessary for an ordinance, but not for a resolution, unless decided otherwise by a majority of all the Sanggunian members.If Congress intended to allow LGUs to exercise eminent domain through a mere resolution, it would have simply adopted the language of the previous Local Government Code. But Congress did not. In a clear divergence from the previous Local

Page 15: pubcorp midterm digests.docx

Government Code, Sec. 19 of R.A. [No.] 7160 categorically requires that the local chief executive act pursuant to an ordinance. x x x 33As respondent’s expropriation in this case was based merely on a resolution, such expropriation is clearly defective. While the Court is aware of the constitutional policy promoting local autonomy, the court cannot grant judicial sanction to an LGU’s exercise of its delegated power of eminent domain in contravention of the very law giving it such power. 34 The Court notes that petitioners failed to raise this point at the earliest opportunity. Still, we are not precluded from considering the same. This Court will not hesitate to consider matters even those raised for the first time on appeal in clearly meritorious situations, 35 such as in this case.Thus, the Court finds it unnecessary to resolve the other issues raised by petitioners.It is well to mention however that despite our ruling in this case respondent is not barred from instituting similar proceedings in the future, provided that it complies with all legal requirements. 36 WHEREFORE, the petition is GRANTED. The decision of the Court of Appeals in CA-G.R. SP No. 47052 is REVERSED and SET ASIDE. The Complaint in Civil Action No. V-6958 is DISMISSED without prejudice.

[G.R. No. 135087. March 14, 2000]HEIRS OF ALBERTO SUGUITAN, petitioner, vs. CITY OF MANDALUYONG, respondent. frnaicsD E C I S I O NGONZAGA_REYES, J.:In this petition for review on certiorari under Rule 45, petitionersi[1] pray for the reversal of the Order dated July 28, 1998 issued by Branch 155 of the Regional Trial Court of Pasig in SCA No. 875 entitled "City of Mandaluyong v. Alberto S. Suguitan, the dispositive portion of which reads as follows:WHEREFORE, in view of the foregoing, the instant Motion to Dismiss is hereby DENIED and an ORDER OF CONDEMNATION is hereby issued declaring that the plaintiff, City of Mandaluyong, has a lawful right to take the subject parcel of land together with existing improvements thereon more specifically covered by Transfer Certificate Of Title No. 56264 of the Registry of Deeds for Metro Manila District II for the public use or purpose as stated in the Complaint, upon payment of just compensation.Accordingly, in order to ascertain the just compensation, the parties are hereby directed to submit to the Court within fifteen (15) days from notice hereof, a list of independent appraisers from which the Court t will select three (3) to be appointed as Commissioners, pursuant to Section 5, Rule 67, Rules of Court.SO ORDERED.ii[2]ellaIt is undisputed by the parties that on October 13, 1994, the Sangguniang Panlungsod of Mandaluyong City issued Resolution No. 396, S-1994iii[3] authorizing then Mayor Benjamin S. Abalos to institute expropriation proceedings over the property of Alberto Sugui located at Boni Avenue and Sto. Rosario streets in Mandaluyong City with an area of 414 square meters and more particularly described under Transfer Certificate of Title No. 56264 of the Registry of Deeds of Metro Manila District II. The intended purpose of the expropriation was the expansion of the Mandaluyong Medical Center.Mayor Benjamin Abalos wrote Alberto Suguitan a letter dated January 20, 1995 offering to buy his property, but Suguitan refused to sell.iv[4] Consequently, on March 13, 1995, the city of Mandaluyong filed a complaintv[5] for expropriation with the Regional Trial Court of Pasig. The case was docketed as SCA No. 875. noveroSuguitan filed a motion to dismissvi[6] the complaint based on the following grounds -(1) the power of eminent domain is not being exercised in accordance with law; (2) there is no public necessity to warrant expropriation of subject property; (3) the City of Mandaluyong seeks to expropriate the said property without payment of just compensation; (4) the City of Mandaluyong has no budget and appropriation for the payment of the property being expropriated; and (5) expropriation of Suguitan' s property is but a ploy of Mayor Benjamin Abalos to acquire the same for his personal use. Respondent filed its comment and opposition to the motion. On October 24, 1995, the trial court denied Suguitan's motion to dismiss.vii[7]On November 14, 1995, acting upon a motion filed by the respondent, the trial court issued an order allowing the City of Mandaluyong to take immediate possession of Suguitan's property upon the deposit of P621,000 representing 15% of the fair market value of the subject property based upon the current tax declaration of such property. On December 15, 1995, the City of Mandaluyong assumed possession of the subject property by virtue of a writ of possession issued by the trial court on December 14, 1995.viii[8] On July 28, 1998, the court granted the assailed order of expropriation.Petitioner assert that the city of Mandaluyong may only exercise its delegated power of eminent domain by means of an ordinance as required by section 19 of Republic Act (RA) No. 7160,ix[9] and not by means of a mere resolution.x[10] Respondent contends, however, that it validly and legally exercised its power of eminent domain; that pursuant to article 36, Rule VI of the Implementing Rules and Regulations (IRR) of RA 7160, a resolution is a sufficient antecedent for the filing

Page 16: pubcorp midterm digests.docx

of expropriation proceedings with the Regional Trial Court. Respondent's position, which was upheld by the trial court, was explained, thus:xi[11]...in the exercise of the respondent City of Mandaluyong's power of eminent domain, a "resolution" empowering the City Mayor to initiate such expropriation proceedings and thereafter when the court has already determine[d] with certainty the amount of just compensation to be paid for the property expropriated, then follows an Ordinance of the Sanggunian Panlungosd appropriating funds for the payment of the expropriated property. Admittedly, title to the property expropriated shall pass from the owner to the expropriator only upon full payment of the just compensation.xii[12] noveroPetitioners refute respondent's contention that only a resolution is necessary upon the initiation of expropriation proceedings and that an ordinance is required only in order to appropriate the funds for the payment of just compensation, explaining that the resolution mentioned in article 36 of the IRR is for purposes of granting administrative authority to the local chief executive to file the expropriation case in court and to represent the local government unit in such case, but does not dispense with the necessity of an ordinance for the exercise of the power of eminent domain under section 19 of the Code.xiii[13]The petition is imbued with merit.Eminent domain is the right or power of a sovereign state to appropriate private property to particular uses to promote public welfare.xiv[14] It is an indispensable attribute of sovereignty; a power grounded in the primary duty of government to serve the common need and advance the general welfare.xv[15] Thus, the right of eminent domain appertains to every independent government without the necessity for constitutional recognition.xvi[16] The provisions found in modern constitutions of civilized countries relating to the taking of property for the public use do not by implication grant the power to the government, but limit a power which would otherwise be without limit.xvii[17] Thus, our own Constitution provides that "[p]rivate property shall not be taken for public use without just compensation."xviii[18] Furthermore, the due process and equal protection clausesxix[19] act as additional safeguards against the arbitrary exercise of this governmental power.Since the exercise of the power of eminent domain affects an individual's right to private property, a constitutionally-protected right necessary for the preservation and enhancement of personal dignity and intimately connected with the rights to life and liberty,xx[20] the need for its circumspect operation cannot be overemphasized. In City of Manila vs. Chinese Community of Manila we said:xxi[21] The exercise of the right of eminent domain, whether directly by the State, or by its authorized agents, is necessarily in derogation of private rights, and the rule in that case is that the authority must be strictly construed. No species of property is held by individuals with greater tenacity, and none is guarded by the constitution and the laws more sedulously, than the right to the freehold of inhabitants. When the legislature interferes with that right, and, for greater public purposes, appropriates the land of an individual without his consent, the plain meaning of the law should not be enlarged by doubt[ful] interpretation. (Bensley vs. Mountainlake Water Co., 13 Cal., 306 and cases cited [73 Am. Dec. 576].)The statutory power of taking property from the owner without his consent is one of the most delicate exercise of governmental authority. It is to be watched with jealous scrutiny. Important as the power may be to the government, the inviolable sanctity which all free constitutions attach to the right of property of the citizens, constrains the strict observance of the substantial provisions of the law which are prescribed as modes of the exercise of the power, and to protect it from abuse. ...(Dillon on Municipal Corporations [5th Ed.], sec. 1040, and cases cited; Tenorio vs. Manila Railroad Co., 22 Phil., 411.)The power of eminent domain is essentially legislative in nature. It is firmly settled, however, that such power may be validly delegated to local government units, other public entities and public utilities, although the scope of this delegated legislative power is necessarily narrower than that of the delegating authority and may only be exercised in strict compliance with the terms of the delegating law.xxii[22] micksThe basis for the exercise of the power of eminent domain by local government units is section 19 of RA 7160 which provides that:A local government unit may, through its chief executive and acting pursuant to an ordinance, exercise the power of eminent domain for public use, purpose, or welfare for the benefits of the poor and the landless, upon payment of just compensation, pursuant to the provisions of the Constitution and pertinent laws; Provided, however, That the power of eminent domain may not be exercised unless a valid and definite offer has been previously made to the owner, and such offer was not accepted; Provided, further, That the local government unit may immediately take possession of the property upon the filing of the expropriation proceedings and upon making a deposit with the proper court of at least fifteen percent (15%) of the fair market value of the property based on the current tax declaration of the property to be expropriated; Provided, finally, That the amount to be paid for the expropriated property shall be determined by the proper court, based on the fair market value at the time of the taking of the property.

Page 17: pubcorp midterm digests.docx

Despite the existence of this legislative grant in favor of local governments, it is still the duty of the courts to determine whether the power of eminent domain is being exercised in accordance with the delegating law.xxiii[23] In fact, the courts have adopted a more censorious attitude in resolving questions involving the proper exercise of this delegated power by local bodies, as compared to instances when it is directly exercised by the national legislature.xxiv[24]The courts have the obligation to determine whether the following requisites have been complied with by the local government unit concerned:1. An ordinance is enacted by the local legislative council authorizing the local chief executive, in behalf of the local government unit, to exercise the power of eminent domain or pursue expropriation proceedings over a particular private property .calr2. The power of eminent domain is exercised for public use, purpose or welfare, or for the benefit of the poor and the landless.3. There is payment of just compensation, as required under Section 9, Article III of the Constitution, and other pertinent laws.4. A valid and definite offer has been previously made to the owner of the property sought to be expropriated, but said offer was not accepted.xxv[25]In the present case, the City of Mandaluyong seeks to exercise the power of eminent domain over petitioners' property by means of a resolution, in contravention of the first requisite. The law in this case is clear and free from ambiguity. Section 19 of the Code requires an ordinance, not a resolution, for the exercise of the power of eminent domain. We reiterate our ruling in Municipality of Parañaque v. V.M. Realty Corporationxxvi[26] regarding the distinction between an ordinance and a resolution. In that 1998 case we held that:misoWe are not convinced by petitioner's insistence that the terms "resolution" and "ordinance" are synonymous. A municipal ordinance is different from a resolution. An ordinance is a law, but a resolution is merely a declaration of the sentiment or opinion of a lawmaking body on a specific matter. An ordinance possesses a general and permanent character, but a resolution is temporary in nature. Additionally, the two are enacted differently -a third reading is necessary for an ordinance, but not for a resolution, unless decided otherwise by a majority of all the Sanggunian members.We cannot uphold respondent's contention that an ordinance is needed only to appropriate funds after the court has determined the amount of just compensation. An examination of the applicable law will show that an ordinance is necessary to authorize the filing of a complaint with the proper court since, beginning at this point, the power of eminent domain is already being exercised.Rule 67 of the 1997 Revised Rules of Court reveals that expropriation proceedings are comprised of two stages:(1) the first is concerned with the determination of the authority of the plaintiff to exercise the power of eminent domain and the propriety of its exercise in the context of the facts involved in the suit; it ends with an order, if not in a dismissal of the action, of condemnation declaring that the plaintiff has a lawful right to take the property sought to be condemned, for the public use or purpose described in the complaint, upon the payment of just compensation to be determined as of the date of the filing of the complaint;(2) the second phase is concerned with the determination by the court of the just compensation for the property sought to be taken; this is done by the court with the assistance of not more than three (3) commissioners.xxvii[27]Clearly, although the determination and award of just compensation to the defendant is indispensable to the transfer of ownership in favor of the plaintiff, it is but the last stage of the expropriation proceedings, which cannot be arrived at without an initial finding by the court that the plaintiff has a lawful right to take the property sought to be expropriated, for the public use or purpose described in the complaint. An order of condemnation or dismissal at this stage would be final, resolving the question of whether or not the plaintiff has properly and legally exercised its power of eminent domain.Also, it is noted that as soon as the complaint is filed the plaintiff shall already have the right to enter upon the possession of the real property involved upon depositing with the court at least fifteen percent (15%) of the fair market value of the property based on the current tax declaration of the property to be expropriated.xxviii[28] Therefore, an ordinance promulgated by the local legislative body authorizing its local chief executive to exercise the power of eminent domain is necessary prior to the filing by the latter of the complaint with the proper court, and not only after the court has determined the amount of just compensation to which the defendant is entitled.basraNeither is respondent's position improved by its reliance upon Article 36 (a), Rule VI of the IRR which provides that:If the LGU fails to acquire a private property for public use, purpose, or welfare through purchase, LGU may expropriate said property through a resolution of the sanggunian authorizing its chief executive to initiate expropriation proceedings.The Court has already discussed this inconsistency between the Code and the IRR, which is more apparent than real, in Municipality of Parañaque vs. V.M. Realty Corporation,xxix[29] which we quote hereunder:

Page 18: pubcorp midterm digests.docx

Petitioner relies on Article 36, Rule VI of the Implementing Rules, which requires only a resolution to authorize an LGU to exercise eminent domain. This is clearly misplaced, because Section 19 of RA 7160, the law itself, surely prevails over said rule which merely seeks to implement it. It is axiomatic that the clear letter of the law is controlling and cannot be amended by a mere administrative rule issued for its implementation. Besides, what the discrepancy seems to indicate is a mere oversight in the wording of the implementing rules, since Article 32, Rule VI thereof, also requires that, in exercising the power of eminent domain, the chief executive of the LGU must act pursuant to an ordinance.Therefore, while we remain conscious of the constitutional policy of promoting local autonomy, we cannot grant judicial sanction to a local government unit's exercise of its delegated power of eminent domain in contravention of the very law giving it such power.It should be noted, however, that our ruling in this case will not preclude the City of Mandaluyong from enacting the necessary ordinance and thereafter reinstituting expropriation proceedings, for so long as it has complied with all other legal requirements.xxx[30]WHEREFORE, the petition is hereby GRANTED. The July 28, 1998 decision of Branch 155 of the Regional Trial Court of Pasig in SCA No. 875 is hereby REVERSED and SET ASIDE.

ASSOCIATION OF SMALL LANDOWNERS IN THE PHILIPPINES, INC., petitionervs.HONORABLE SECRETARY OF AGRARIAN REFORM, respondent.

G.R. No. 78742 July 14, 1989

"Land for the Landless" is a slogan that underscores the acute imbalance in the distribution of this precious resource among our people. But it is more than a slogan. Through the brooding centuries, it has become a battle-cry dramatizing the increasingly urgent demand of the dispossessed among us for a plot of earth as their place in the sun.

Recognizing this need, the Constitution in 1935 mandated the policy of social justice to "insure the well-being and economic security of all the people," especially the less privileged. In 1973, the new Constitution affirmed this goal adding specifically that "the State shall regulate the acquisition, ownership, use, enjoyment and disposition of private property and equitably diffuse property ownership and profits." Significantly, there was also the specific injunction to "formulate and implement an agrarian reform program aimed at emancipating the tenant from the bondage of the soil."

Facts:

The petitioners in this case invoke the right of retention granted by P.D. No. 27 to owners of rice and corn lands not exceeding seven hectares as long as they are cultivating or intend to cultivate the same. Their respective lands do not exceed the statutory limit but are occupied by tenants who are actually cultivating such lands.

According to P.D. No. 316, which was promulgated in implementation of P.D. No. 27:

No tenant-farmer in agricultural lands primarily devoted to rice and corn shall be ejected or removed from his farmholding until such time as the respective rights of the tenant- farmers and the landowner shall have been determined in accordance with the rules and regulations implementing P.D. No. 27.

The petitioners claim they cannot eject their tenants and so are unable to enjoy their right of retention because the Department of Agrarian Reform has so far not issued the implementing rules required under the above-quoted decree. They therefore ask the Court for a writ of mandamus to compel the respondent to issue the said rules.

The public respondent argues that P.D. No. 27 has been amended by LOI 474 removing any right of retention from persons who own other agricultural lands of more than 7 hectares in aggregate area or lands used for residential, commercial, industrial or other purposes from which they derive adequate income for their family. And even assuming that the petitioners do not fall under its terms, the regulations implementing P.D. No. 27 have already been issued, to wit, the Memorandum dated July 10, 1975 (Interim Guidelines on Retention by Small Landowners, with an accompanying Retention

Page 19: pubcorp midterm digests.docx

Guide Table), Memorandum Circular No. 11 dated April 21, 1978, (Implementation Guidelines of LOI No. 474), Memorandum Circular No. 18-81 dated December 29,1981 (Clarificatory Guidelines on Coverage of P.D. No. 27 and Retention by Small Landowners), and DAR Administrative Order No. 1, series of 1985 (Providing for a Cut-off Date for Landowners to Apply for Retention and/or to Protest the Coverage of their Landholdings under Operation Land Transfer pursuant to P.D. No. 27). For failure to file the corresponding applications for retention under these measures, the petitioners are now barred from invoking this right.

The petitioners insist that the above-cited measures are not applicable to them because they do not own more than seven hectares of agricultural land.

The Constitution of 1987 was not to be outdone. Besides echoing these sentiments, it also adopted one whole and separate Article XIII on Social Justice and Human Rights, containing grandiose but undoubtedly sincere provisions for the uplift of the common people. These include a call in the following words for the adoption by the State of an agrarian reform program:

SEC. 4. The State shall, by law, undertake an agrarian reform program founded on the right of farmers and regular farmworkers, who are landless, to own directly or collectively the lands they till or, in the case of other farmworkers, to receive a just share of the fruits thereof. To this end, the State shall encourage and undertake the just distribution of all agricultural lands, subject to such priorities and reasonable retention limits as the Congress may prescribe, taking into account ecological, developmental, or equity considerations and subject to the payment of just compensation. In determining retention limits, the State shall respect the right of small landowners. The State shall further provide incentives for voluntary land-sharing.

Issue:

Whether or not all rights acquired by the tenant-farmer under P.D. No. 27, as recognized under E.O. No. 228, are retained by him even under R.A. No. 6657.

Held:

P.D. No. 27 expressly ordered the emancipation of tenant-farmer as October 21, 1972 and declared that he shall "be deemed the owner" of a portion of land consisting of a family-sized farm except that "no title to the land owned by him was to be actually issued to him unless and until he had become a full-fledged member of a duly recognized farmers' cooperative." It was understood, however, that full payment of the just compensation also had to be made first, conformably to the constitutional requirement.

When E.O. No. 228, categorically stated in its Section 1 that:

All qualified farmer-beneficiaries are now deemed full owners as of October 21, 1972 of the land they acquired by virtue of Presidential Decree No. 27.

The CARP Law, for its part, conditions the transfer of possession and ownership of the land to the government on receipt by the landowner of the corresponding payment or the deposit by the DAR of the compensation in cash or LBP bonds with an accessible bank. Until then, title also remains with the landowner. No outright change of ownership is contemplated either.

This should counter-balance the express provision in Section 6 of the said law that "the landowners whose lands have been covered by Presidential Decree No. 27 shall be allowed to keep the area originally retained by them thereunder, further, that original homestead grantees or direct compulsory heirs who still own the original homestead at the time of the approval of this Act shall retain the same areas as long as they continue to cultivate said homestead."

R.A. No. 6657 does provide for such limits now in Section 6 of the law, which in fact is one of its most controversial provisions.

Retention Limits. — Except as otherwise provided in this Act, no person may own or retain, directly or indirectly, any public or private agricultural land, the size of which shall vary according to factors governing a viable family-sized farm, such as

Page 20: pubcorp midterm digests.docx

commodity produced, terrain, infrastructure, and soil fertility as determined by the Presidential Agrarian Reform Council (PARC) created hereunder, but in no case shall retention by the landowner exceed five (5) hectares. Three (3) hectares may be awarded to each child of the landowner, subject to the following qualifications: (1) that he is at least fifteen (15) years of age; and (2) that he is actually tilling the land or directly managing the farm; Provided, That landowners whose lands have been covered by Presidential Decree No. 27 shall be allowed to keep the area originally retained by them thereunder, further, That original homestead grantees or direct compulsory heirs who still own the original homestead at the time of the approval of this Act shall retain the same areas as long as they continue to cultivate said homestead.

All rights previously acquired by the tenant- farmers under P.D. No. 27 are retained and recognized. Landowners who were unable to exercise their rights of retention under P.D. No. 27 shall enjoy the retention rights granted by R.A. No. 6657 under the conditions therein prescribed. Subject to the above-mentioned rulings all the petitions are DISMISSED, without pronouncement as to costs.

Primicias vs. FugosoIn the case of Primicias vs. Fugoso, G.R. No. L-1800 January 27, 1948, the Supreme Court said that fear that trouble may arise during the rally was not enough reason to suppress the fundamental right of the people to free speech and peaceful assembly to petition the government for redress of grievances.

FactsIn November 1947, the mayor of Manila refused to grant a permit to hold a public meeting at Plaza Miranda for the purpose of petitioning the government for redress of grievances. The mayor’s reason: "that there is a reasonable ground to believe, basing upon previous utterances and upon the fact that passions, specially on the part of the losing groups, remains bitter and high, that similar speeches will be delivered tending to undermine the faith and confidence of the people in their government, and in the duly constituted authorities, which might threaten breaches of the peace and a disruption of public order." A Manila ordinance at that time required a mayor’s permit to hold a parade or procession, or, by analogy, a public meeting or assembly. Primicias filed a case to compel the mayor to grant the permit.

[edit] IssueMay the mayor be compelled to grant the permit? [edit] RulingYes. The Supreme Court ordered the mayor to grant the permit, interpreting the ordinance to mean that the Mayor did not have the power to grant or refuse the permit, only the discretion to specify where the parade may pass or where the meeting may be held. The Court said that the constitutional right to free speech and peaceful assembly was a fundamental right of the people and may not be suppressed unless there was the probability of serious injury to the state, and quoted US Supreme Court Justice Brandeis in Whitney vs. California: "Fear of serious injury cannot alone justify suppression of free speech and assembly.”

Primicias vs. Fugoso [L-18000. Jan 27, 1948]Doctrine:Clear and Present Danger Test, Freedom of Assembly and ExpressionFACTS:This case is an action of mandamus instituted by petitioner Cipriano Primicias, manager of theCoalesced Minority Parties, against respondent Manila City Mayor, Valeriano Fugoso, to compel thelatter to issue a permit for the holding of a public meeting at the Plaza Miranda on Nov 16, 1947. Thepetitioner requested for a permit to hold a “peaceful public meeting”.However, the respondent refusedto issue such permit because he found “that there is a reasonable ground to believe, basing uponprevious utterances and upon the fact that passions, specially on the part of the losing groups, remainsbitter and high, that similar speeches will be delivered tending to undermine the faith and confidence of the people in their government, and in the duly peace and a disruption of public order.”

Page 21: pubcorp midterm digests.docx

Respondentbased his refusal to the Revised Ordinances of 1927 prohibiting as an offense against public peace, andpenalizes as a misdemeanor, "any act, in any public place, meeting, or procession, tending to disturb thepeace or excite a riot; or collect with other persons in a body or crowd for any unlawful purpose; ordisturb or disquiet any congregation engaged in any lawful assembly." Included herein is Sec. 1119, Freeuse of Public Place.ISSUE:Whether or not the Mayor has the right to refuse to issue permit hence violating freedom of assembly.HELD:The answer is negative. Supreme Court states that the freedom of speech, and to peacefullyassemble and petition the government for redress of grievances, are fundamental personal rights of thepeople recognized and guaranteed by the constitution. However, these rights are not absolute. They canbe regulated under the state’s police power –that they should not be injurious to the equal enjoymentof others having equal rights, nor to the rights of the community or society.The Court holds that there can be 2 interpretations of Sec. 1119: 1) the Mayor of the City of Manila isvested with unregulated discretion to grant or refuse, to grant permit for the holding of a lawfulassembly or meeting, parade, or procession in the streets and other public places of the City of Manila;and 2) The right of the Mayor is subject to reasonable discretion to determine or specify the streets orpublic places to be used with the view to prevent confusion by overlapping, to secure convenient use of the streets and public places by others, and to provide adequate and proper policing to minimize therisk of disorder.The court favored the second construction since the first construction is tantamount to authorizing theMayor to prohibit the use of the streets. Under our democratic system of government no such unlimitedpower may be validly granted to any officer of the government, except perhaps in cases of nationalemergency. It is to be noted that the permit to be issued is for the use of public places and not for theassembly itself.The Court holds that the assembly is lawful and thus cannot be struck down. Fear of serious injurycannot alone justify suppression of free speech and assembly. It is the function of speech to free menfrom the bondage of irrational fears. To justify suppression of free speech there must be reasonableground to fear that serious evil will result if free speech is practiced. There must be reasonable groundto believe that the danger apprehended is imminent. There must be reasonable ground to believe thatthe evil to be prevented is a serious one . The fact that speech is likely to result in some violence or indestruction of property is not enough to justify its suppression. There must be the probability of seriousinjury to the state.PETITION IS GRANTED

Laguna Lake Development Authority vs. Court of Appeals(December 7, 1995)Ponentia: Hermosisima, Jr., J.Facts:1.Republic Act No. 4850created the “Laguna Lake Development Authority” (Authority) – a Government Agency that works toward environmentalprotection and ecology, navigational safety, and sustainable development. Thisagency is responsible for the development of the Laguna Lake area and thesurrounding provinces, cities and towns in view of the national and regionalplans.2.President Ferdinand E. Marcos then passedPresidential Decree No. 813amending certain sections of R.A. No. 4850as response to the deterioratingenvironmental condition of the Metropolitan Manila area and the surroundingareas of the Laguna de Bay. Problems include the environmental impact of development of water quality, inflow of polluted water, increasing urbanizationand floods in Metropolitan Manila.3.Sec. 1 of P.D. 813established a policy of development with environmentalmanagement and control, among others for the Laguna Lake DevelopmentAuthority. Special powers, pertinent to this case, were also granted underSec.3. which include the exclusive jurisdiction

Page 22: pubcorp midterm digests.docx

of the Authority toissue new permit for the use of the lake waters for any projects or activities in or affectingthe said lake including navigation, construction, and 4.operation of fishpens, fish enclosures, fish corrals and the like. The Authorityalso has the power to collect fees for these activities and projects which may beshared with other governmental agencies and political sub-divisions.5.The Authority was further empowered byExecutive Order No. 927whichenlarged its functions and powers. Said Order also named and enumeratedtowns, cities and provinces encompassed by the term “Laguna de Bay Region.” The Chief Executive based this Order on an assessment that the land andwaters of the Laguna Lake Region are limited natural resources requiring judicious management.6.Under Sec. 2 of E.O. 927, the Authority shall have exclusive jurisdiction toissue permit for the use of all surface water for any projects or activities in oraffecting the said region. Coverage for “Laguna de Bay Region” included severalprovinces, cities and towns around the Laguna Lake. UnderSec. 3, thecollection of fees for the use of the lake water and its tributaries were enforcedby the Authority.7.Then,Republic Act No. 7160,the Local Government Code of 1991 wasenforced. Municipalities around the Laguna Lake Region interpreted this law asdelegating the exclusive jurisdiction to issue fishing privileges within theirmunicipal waters.8.Municipal governments started issuing fishing privileges and fishing permits tobig fishpen operators. These unregulated issuances of Mayor’s permits toconstruct fishpens were clear violations of the policies implemented by theAuthority.9.To solve the problem, the Authority issues a notice to the general publicdeclaring as illegal all fishpens, fishcages and other aqua-culture structures inthe Bay Region that were not registered with the Authority. The notice includesa threat of penalty of demolition and imprisonment and/or fine. After a month the Authority sent notices to the concerned owners stating that demolition shallbe effected within 10 days.10.Affected fishpen owners filed seven injunction cases against the Authority invarious trial courts. Authority filed a motion to dismiss the cases on jurisdictional grounds which was denied by the lower court. Temporaryrestraining order/writs of preliminary injunction was issued enjoining theAuthority from demolishing the structures in question. Authority appealed thecase to the Court of Appeals but the Court dismissed the consolidated petitionsof the Authority. CA established that LLDA is not a quasi-judicial agency of thegovernment and it cannot exercise quasi-judicial functions as far as fishpensare concerned. CA the Local Government Code of 1991 repealed the provisionsof the LLDA Charter thereby devolving the power to grant permits to the localgovernment units concerned. Authority appealed to the Supreme Court withpetitions for prohibition, certiorari and injunction against the respondents.Issue:WON the Laguna Lake Development Authority should exercise jurisdictionover the Laguna Lake insofar as the issuance of permits for fisheries privileges isconcerned.Held: Yes.The LLDA should exercise jurisdiction over the Laguna Lake insofar asthe issuance of permits for fisheries is concerned.Petitions for prohibition, certiorari and injunction are hereby granted,insofar as they related to the authority of the LLDA to grant fishing privileges withinthe Laguna Lake Region.Restraining orders and/or writs of injunction issued against the LLDA aredeclared null and void and ordered set aside for having been issued with graveabuse of discretion.Municipal Mayors of the Laguna Lake Region are hereby prohibited fromissuing permits to construct and operate fishpens, fishcages and other aqua-culturestructures within the Laguna Lake Region. Previous issuances are null and void.The fishpens, fishcages and other aqua-culture structures put by operatorsby virtue of permits issued by Municipal Mayors within the Laguna Lake Region arehereby declared illegal structures subject to demolition by the LLDA.Ratio:(1)Provisions of the Local Government Code of 1991 (RA No. 7160) do not repealthe laws creating the LLDA. Therefore, LLDA maintains its exclusive authorityover issuances of permits.-

Page 23: pubcorp midterm digests.docx

The charter of the LLDA is a SPECIAL LAW while the Local Government Code of 1991 is a GENERAL LAW. A basic rule of statutory construction is that theenactment of a later legislation which is a general law cannot be construed tohave repealed a special law.-When there is conflict between a general law and a special law, the special lawwill prevail since it evinces the legislative intent more clearly than the generalstatute.-A special law cannot be repealed, amended or altered by a subsequent law bymere implications.-InManila Railroad Company v. Rafferty , this basic rule is upheld with anexception when the subsequent general law has a manifest intent to repeal oralter the special law. (2)Legislative intent is for the Authority to proceed with its mission of environmental protection, navigational safety, and sustainable development forthe Laguna Lake Region.-The power of the local government units, exercised through fragmentedmanagement policies, is interested in fishing privileges for REVENUEPURPOSES. In contrast, the power of the Authority is aimed at effectivelyregulating and monitoring activities in the Laguna de Bay Region for QUALITYCONTROL and MANAGEMENT. Thus, the Authority is in a better position tomanage the issuance of permits.(3)Charter of the Authority prevails because it is a valid exercise of POLICE POWERof the State.(4)Although the LLDA is not co-equal to the RTCs, LLDA is still a quasi-judicialbody with respect to pollution cases that can issue ‘cease and desist order’ (Laguna Lake Development Authority v. CA).Padilla, J. (Concurring Opinion):I fully concur. But I just want to say guys that the LGUs can still impose on thosewho apply for permit with an additional local permit or license for revenue purposes.This would harmonize RA No. 4850 withRA No. 7160(LGC 1991)In this case, such intent is not proven in this case

[G.R. No. 148622. September 12, 2002]REPUBLIC OF THE PHILIPPINES, represented by HON. HEHERSON T. ALVAREZ, in his capacity as Secretary of the DEPARTMENT OF ENVIRONMENT AND NATURAL RESOURCES (DENR), CLARENCE L. BAGUILAT, in his capacity as the Regional Executive Director of DENR-Region XI and ENGR. BIENVENIDO L. LIPAYON, in his capacity as the Regional Director of the DENR-ENVIRONMENTAL MANAGEMENT BUREAU (DENR-EMB), Region XI, petitioners, vs. THE CITY OF DAVAO, represented by BENJAMIN C. DE GUZMAN, City Mayor, respondent.D E C I S I O NYNARES-SANTIAGO, J.:Before us is a petition for reviewxxxi[1] on certiorari assailing the decisionxxxii[2] dated May 28, 2001 of the Regional Trial Court of Davao City, Branch 33, which granted the writ of mandamus and injunction in favor of respondent, the City of Davao, and against petitioner, the Republic, represented by the Department of Environment and Natural Resources (DENR). The trial court also directed petitioner to issue a Certificate of Non-Coverage in favor of respondent.The antecedent facts of the case are as follows:On August 11, 2000, respondent filed an application for a Certificate of Non-Coverage (CNC) for its proposed project, the Davao City Artica Sports Dome, with the Environmental Management Bureau (EMB), Region XI. Attached to the application were the required documents for its issuance, namely, a) detailed location map of the project site; b) brief project description; and c) a certification from the City Planning and Development Office that the project is not located in an

Page 24: pubcorp midterm digests.docx

environmentally critical area (ECA). The EMB Region XI denied the application after finding that the proposed project was within an environmentally critical area and ruled that, pursuant to Section 2, Presidential Decree No. 1586, otherwise known as the Environmental Impact Statement System, in relation to Section 4 of Presidential Decree No, 1151, also known as the Philippine Environment Policy, the City of Davao must undergo the environmental impact assessment (EIA) process to secure an Environmental Compliance Certificate (ECC), before it can proceed with the construction of its project. Believing that it was entitled to a Certificate of Non-Coverage, respondent filed a petition for mandamus and injunction with the Regional Trial Court of Davao, docketed as Civil Case No. 28,133-2000. It alleged that its proposed project was neither an environmentally critical project nor within an environmentally critical area; thus it was outside the scope of the EIS system. Hence, it was the ministerial duty of the DENR, through the EMB-Region XI, to issue a CNC in favor of respondent upon submission of the required documents.The Regional Trial Court rendered judgment in favor of respondent, the dispositive portion of which reads as follows:WHEREFORE, finding the petition to be meritorious, judgment granting the writ of mandamus and injunction is hereby rendered in favor of the petitioner City of Davao and against respondents Department of Environment and Natural Resources and the other respondents by:1) directing the respondents to issue in favor of the petitioner City of Davao a Certificate of Non-Coverage, pursuant to Presidential Decree No. 1586 and related laws, in connection with the construction by the City of Davao of the Artica Sports Dome;2) making the preliminary injunction issued on December 12, 2000 permanent.Costs de oficio.SO ORDERED.xxxiii[3]The trial court ratiocinated that there is nothing in PD 1586, in relation to PD 1151 and Letter of Instruction No. 1179 (prescribing guidelines for compliance with the EIA system), which requires local government units (LGUs) to comply with the EIS law. Only agencies and instrumentalities of the national government, including government owned or controlled corporations, as well as private corporations, firms and entities are mandated to go through the EIA process for their proposed projects which have significant effect on the quality of the environment. A local government unit, not being an agency or instrumentality of the National Government, is deemed excluded under the principle of expressio unius est exclusio alterius. The trial court also declared, based on the certifications of the DENR-Community Environment and Natural Resources Office (CENRO)-West, and the data gathered from the Philippine Institute of Volcanology and Seismology (PHIVOLCS), that the site for the Artica Sports Dome was not within an environmentally critical area. Neither was the project an environmentally critical one. It therefore becomes mandatory for the DENR, through the EMB Region XI, to approve respondent’s application for CNC after it has satisfied all the requirements for its issuance. Accordingly, petitioner can be compelled by a writ of mandamus to issue the CNC, if it refuses to do so.Petitioner filed a motion for reconsideration, however, the same was denied. Hence, the instant petition for review.With the supervening change of administration, respondent, in lieu of a comment, filed a manifestation expressing its agreement with petitioner that, indeed, it needs to secure an ECC for its proposed project. It thus rendered the instant petition moot and academic. However, for the guidance of the implementors of the EIS law and pursuant to our symbolic function to educate the bench and bar,xxxiv[4] we are inclined to address the issue raised in this petition.Section 15 of Republic Act 7160,xxxv[5] otherwise known as the Local Government Code, defines a local government unit as a body politic and corporate endowed with powers to be exercised by it in conformity with law. As such, it performs dual functions, governmental and proprietary. Governmental functions are those that concern the health, safety and the advancement of the public good or welfare as affecting the public generally.xxxvi[6] Proprietary functions are those that seek to obtain special corporate benefits or earn pecuniary profit and intended for private advantage and benefit.xxxvii[7] When exercising governmental powers and performing governmental duties, an LGU is an agency of the national government.xxxviii

[8] When engaged in corporate activities, it acts as an agent of the community in the administration of local affairs.xxxix[9]Found in Section 16 of the Local Government Code is the duty of the LGUs to promote the people’s right to a balanced ecology.xl[10] Pursuant to this, an LGU, like the City of Davao, can not claim exemption from the coverage of PD 1586. As a body politic endowed with governmental functions, an LGU has the duty to ensure the quality of the environment, which is the very same objective of PD 1586.Further, it is a rule of statutory construction that every part of a statute must be interpreted with reference to the context, i.e., that every part must be considered with other parts, and kept subservient to the general intent of the enactment.xli[11] The trial court, in declaring local government units as exempt from the coverage of the EIS law, failed to relate Section 2 of PD 1586xlii[12] to the following provisions of the same law:

Page 25: pubcorp midterm digests.docx

WHEREAS, the pursuit of a comprehensive and integrated environmental protection program necessitates the establishment and institutionalization of a system whereby the exigencies of socio-economic undertakings can be reconciled with the requirements of environmental quality; x x x.Section 1. Policy. – It is hereby declared the policy of the State to attain and maintain a rational and orderly balance between socio-economic growth and environmental protection.x x x x x x x x xSection 4. – Presidential Proclamation of Environmentally Critical Areas and Projects. – The President of the Philippines may, on his own initiative or upon recommendation of the National Environmental Protection Council, by proclamation declare certain projects, undertakings or areas in the country as environmentally critical. No person, partnership or corporation shall undertake or operate any such declared environmentally critical project or area without first securing an Environmental Compliance Certificate issued by the President or his duly authorized representative. For the proper management of said critical project or area, the President may by his proclamation reorganize such government offices, agencies, institutions, corporations or instrumentalities including the realignment of government personnel, and their specific functions and responsibilities.Section 4 of PD 1586 clearly states that “no person, partnership or corporation shall undertake or operate any such declared environmentally critical project or area without first securing an Environmental Compliance Certificate issued by the President or his duly authorized representative.”xliii[13] The Civil Code defines a person as either natural or juridical. The state and its political subdivisions, i.e., the local government unitsxliv[14] are juridical persons.xlv[15] Undoubtedly therefore, local government units are not excluded from the coverage of PD 1586.Lastly, very clear in Section 1 of PD 1586 that said law intends to implement the policy of the state to achieve a balance between socio-economic development and environmental protection, which are the twin goals of sustainable development. The above-quoted first paragraph of the Whereas clause stresses that this can only be possible if we adopt a comprehensive and integrated environmental protection program where all the sectors of the community are involved, i.e., the government and the private sectors. The local government units, as part of the machinery of the government, cannot therefore be deemed as outside the scope of the EIS system.xlvi[16]The foregoing arguments, however, presuppose that a project, for which an Environmental Compliance Certificate is necessary, is environmentally critical or within an environmentally critical area. In the case at bar, respondent has sufficiently shown that the Artica Sports Dome will not have a significant negative environmental impact because it is not an environmentally critical project and it is not located in an environmentally critical area. In support of this contention, respondent submitted the following:1. Certification from the City Planning and Development Office that the project is not located in an environmentally critical area;2. Certification from the Community Environment and Natural Resources Office (CENRO-West) that the project area is within the 18-30% slope, is outside the scope of the NIPAS (R.A. 7586), and not within a declared watershed area; and3. Certification from PHILVOCS that the project site is thirty-seven (37) kilometers southeast of the southernmost extension of the Davao River Fault and forty-five (45) kilometers west of the Eastern Mindanao Fault; and is outside the required minimum buffer zone of five (5) meters from a fault zone.The trial court, after a consideration of the evidence, found that the Artica Sports Dome is not within an environmentally critical area. Neither is it an environmentally critical project. It is axiomatic that factual findings of the trial court, when fully supported by the evidence on record, are binding upon this Court and will not be disturbed on appeal.xlvii[17] This Court is not a trier of facts.xlviii[18]There are exceptional instances when this Court may disregard factual findings of the trial court, namely: a) when the conclusion is a finding grounded entirely on speculations, surmises, or conjectures; b) when the inference made is manifestly mistaken, absurd, or impossible; c) where there is a grave abuse of discretion; d) when the judgment is based on a misapprehension of facts; e) when the findings of fact are conflicting; f) when the Court of Appeals, in making its findings, went beyond the issues of the case and the same are contrary to the admissions of both appellant and appellee; g) when the findings of the Court of Appeals are contrary to those of the trial court; h) when the findings of fact are conclusions without citation of specific evidence on which they are based; i) when the finding of fact of the Court of Appeals is premised on the supposed absence of evidence but is contradicted by the evidence on record; and j) when the Court of Appeals manifestly overlooked certain relevant facts not disputed by the parties and which, if properly considered, would justify a different conclusion.xlix[19] None of these exceptions, however, obtain in this case.The Environmental Impact Statement System, which ensures environmental protection and regulates certain government activities affecting the environment, was established by Presidential Decree No. 1586. Section 2 thereof states:

Page 26: pubcorp midterm digests.docx

There is hereby established an Environmental Impact Statement System founded and based on the environmental impact statement required under Section 4 of Presidential Decree No. 1151, of all agencies and instrumentalities of the national government, including government-owned or controlled corporations, as well as private corporations, firms and entities, for every proposed project and undertaking which significantly affect the quality of the environment.Section 4 of PD 1151, on the other hand, provides:Environmental Impact Statements. – Pursuant to the above enunciated policies and goals, all agencies and instrumentalities of the national government, including government-owned or controlled corporations, as well as private corporations, firms and entities shall prepare, file and include in every action, project or undertaking which significantly affects the quality of the environment a detailed statement on–(a) the environmental impact of the proposed action, project or undertaking (b) any adverse environmental effect which cannot be avoided should the proposal be implemented(c) alternative to the proposed action(d) a determination that the short-term uses of the resources of the environment are consistent with the maintenance and enhancement of the long-term productivity of the same; and(e) whenever a proposal involves the use of depletable or nonrenewable resources, a finding must be made that such use and commitment are warranted.Before an environmental impact statement is issued by a lead agency, all agencies having jurisdiction over, or special expertise on, the subject matter involved shall comment on the draft environmental impact statement made by the lead agency within thirty (30) days from receipt of the same.Under Article II, Section 1, of the Rules and Regulations Implementing PD 1586, the declaration of certain projects or areas as environmentally critical, and which shall fall within the scope of the Environmental Impact Statement System, shall be by Presidential Proclamation, in accordance with Section 4 of PD 1586 quoted above.Pursuant thereto, Proclamation No. 2146 was issued on December 14, 1981, proclaiming the following areas and types of projects as environmentally critical and within the scope of the Environmental Impact Statement System established under PD 1586:A. Environmentally Critical ProjectsI. Heavy Industriesa. Non-ferrous metal industriesb. Iron and steel millsc. Petroleum and petro-chemical industries including oil and gasd. Smelting plantsII. Resource Extractive Industriesa. Major mining and quarrying projectsb. Forestry projects1. Logging2. Major wood processing projects3. Introduction of fauna (exotic-animals) in public/private forests4. Forest occupancy5. Extraction of mangrove products6. Grazingc. Fishery Projects1. Dikes for/and fishpond development projectsIII. Infrastructure Projectsa. Major damsb. Major power plants (fossil-fueled, nuclear fueled, hydroelectric or geothermal)c. Major reclamation projectsd. Major roads and bridgesB. Environmentally Critical Areas1. All areas declared by law as national parks, watershed reserves, wildlife preserves and sanctuaries;2. Areas set aside as aesthetic potential tourist spots;3. Areas which constitute the habitat for any endangered or threatened species of indigenous Philippine Wildlife (flora and fauna);4. Areas of unique historic, archaeological, or scientific interests;5. Areas which are traditionally occupied by cultural communities or tribes;

Page 27: pubcorp midterm digests.docx

6. Areas frequently visited and/or hard-hit by natural calamities (geologic hazards, floods, typhoons, volcanic activity, etc.);7. Areas with critical slopes;8. Areas classified as prime agricultural lands;9. Recharged areas of aquifers;10. Water bodies characterized by one or any combination of the following conditions;a. tapped for domestic purposesb. within the controlled and/or protected areas declared by appropriate authoritiesc. which support wildlife and fishery activities11. Mangrove areas characterized by one or any combination of the following conditions:a. with primary pristine and dense young growth;b. adjoining mouth of major river systems;c. near or adjacent to traditional productive fry or fishing grounds;d. which act as natural buffers against shore erosion, strong winds and storm floods;e. on which people are dependent for their livelihood.12. Coral reefs, characterized by one or any combinations of the following conditions:a. with 50% and above live coralline cover;b. spawning and nursery grounds for fish;c. which act as natural breakwater of coastlines.In this connection, Section 5 of PD 1586 expressly states:Environmentally Non-Critical Projects. — All other projects, undertakings and areas not declared by the President as environmentally critical shall be considered as non-critical and shall not be required to submit an environmental impact statement. The National Environmental Protection Council, thru the Ministry of Human Settlements may however require non-critical projects and undertakings to provide additional environmental safeguards as it may deem necessary. The Artica Sports Dome in Langub does not come close to any of the projects or areas enumerated above. Neither is it analogous to any of them. It is clear, therefore, that the said project is not classified as environmentally critical, or within an environmentally critical area. Consequently, the DENR has no choice but to issue the Certificate of Non-Coverage. It becomes its ministerial duty, the performance of which can be compelled by writ of mandamus, such as that issued by the trial court in the case at bar.WHEREFORE, in view of the foregoing, the instant petition is DENIED. The decision of the Regional Trial Court of Davao City, Branch 33, in Civil Case No. 28,133-2000, granting the writ of mandamus and directing the Department of Environment and Natural Resources to issue in favor of the City of Davao a Certificate of Non-Coverage, pursuant to Presidential Decree No. 1586 and related laws, in connection with the construction of the Artica Sports Dome, is AFFIRMED.

Lino Vs Pano

HON. JOSE D. LINA, JR., SANGGUNIANG PANLALAWIGAN OF LAGUNA, and HON.CALIXTO CATAQUIZ, petitioners, vs. HON. FRANCISCO DIZON PAÑO and TONYCALVENTO,respondents.G.R. No. 129093

FACTS:On December 29, 1995, respondent Tony Calvento was appointed agent by the Philippine Charity Sweepstakes Office (PCSO) to install Terminal OM 20 for the operation of lotto. He asked Mayor Calixto Cataquiz, Mayor of San Pedro, Laguna, for a mayor’s permit to open the lotto outlet. This was denied by Mayor Cataquiz in a letter dated February 19, 1996. The ground for said denial was an ordinance passed by the Sangguniang Panlalawigan of Laguna entitled Kapasiyahan Blg. 508, T. 1995which was issued on September 18, 1995.As a result of this resolution of denial, respondent Calvento filed a complaint for declaratory relief with prayer for preliminary injunction and temporary restraining order. In the said complaint, respondent Calvento asked the Regional Trial Court of San Pedro Laguna, Branch 93, for the following reliefs: (1) a preliminary injunction or temporary restraining order, ordering the defendants to refrain from implementing or enforcing Kapasiyahan Blg. 508, T. 1995; (2) an order requiring Hon. Municipal Mayor Calixto R. Cataquiz to issue a business permit for the operation of a lotto outlet; and (3) an order annulling or declaring as invalid Kapasiyahan Blg. 508, T. 1995.On

Page 28: pubcorp midterm digests.docx

February 10, 1997, the respondent judge, Francisco Dizon Paño, promulgated his decision enjoining the petitioners from implementing or enforcing resolution or Kapasiyahan Blg. 508, T. 1995.

ISSUE: WON Kapasiyahan Blg. 508, T. 1995 is valid

HELD: As a policy statement expressing the local government’s objection to the lotto, such resolution is valid. This is part of the local government’s autonomy to air its views which may be contrary to that of the national government’s. However, this freedom to exercise contrary views does not mean that local governments may actually enact ordinances that go against laws duly enacted by Congress. Given this premise, the assailed resolution in this case could not and should not be interpreted as a measure or ordinance prohibiting the operation of lotto.n our system of government, the power of local government units to legislate and enact ordinances and resolutions is merely a delegated power coming from Congress. As held in Tatel vs. Virac, ordinances should not contravene an existing statute enacted by Congress. The reasons for this is obvious, as elucidated in Magtajas v. Pryce Properties Corp

G.R. No. 111097 July 20, 1994MAYOR PABLO P. MAGTAJAS & THE CITY OF CAGAYAN DE ORO, petitioners, vs.PRYCE PROPERTIES CORPORATION, INC. & PHILIPPINE AMUSEMENT AND GAMING CORPORATION,

FACTS: There was instant opposition when PAGCOR announced the opening of a casino in Cagayan de Oro City. Civic organizations angrily denounced the project.The trouble arose when in 1992, flush with its tremendous success in several cities, PAGCOR decided to expand its operations to Cagayan de Oro City.he reaction of the Sangguniang Panlungsod of Cagayan de Oro City was swift and hostile. On December 7, 1992, it enacted Ordinance No. 3353.Nor was this all. On January 4, 1993, it adopted a sterner Ordinance No. 3375-93Pryce assailed the ordinances before the Court of Appeals, where it was joined by PAGCOR as intervenor and supplemental petitioner. Their challenge succeeded. On March 31, 1993, the Court of Appeals declared the ordinances invalid and issued the writ prayed for to prohibit their enforcement

ISSUE: WON Ordinance 3353 and 3375-93 valid

HELD: NoLocal Government Code, local government units are authorized to prevent or suppress, among others, "gambling and other prohibited games of chance." Obviously, this provision excludes games of chance which are not prohibited but are in fact permitted by law.The rationale of the requirement that the ordinances should not contravene a statute is obvious.Casino gambling is authorized by P.D. 1869. This decree has the status of a statute that cannot be amended or nullified by a mere ordinance. Hence, it was not competent for the Sangguniang Panlungsod of Cagayan de Oro City to enact Ordinance No. 3353 prohibiting the use of buildings for the operation of a casino and Ordinance No. 3375-93 prohibiting the operation of casinos. For all their praiseworthy motives, these ordinances are contrary to P.D. 1869 and the public policy announced therein and are therefore ultra vires and void.

Page 29: pubcorp midterm digests.docx

i

ii

iii

iv

v

vi

vii

viii

ix

x

xi

xii

xiii

xiv

xv

xvi

xvii

xviii

xix

xx

xxi

xxii

xxiii

xxiv

xxv

xxvi

xxvii

Page 30: pubcorp midterm digests.docx

xxviii

xxix

xxx

xxxi

xxxii

xxxiii

xxxiv

xxxv

xxxvi

xxxvii

xxxviii

xxxix

xl

xli

xlii

xliii

xliv

xlv

xlvi

xlvii

xlviii

xlix